Aplitude
Aplitude
BASIC FORMULAS:
=(2a+b-c)
+1 - (-1)
To subtract a negative number, you can change the sign and convert it to addition:
+1 + 1
Question 3 : What would be the maximum value of 'B' in the following equation :
12B
+B4C
+C67
--------
1035
--------
Solution : Only the leftmost part of the number can be of two or more digits. So, we split the answer
as :
12B
+B4C
+C67
--------
10 3 5
--------
128
+840
+067
--------
10 3 5
--------
Question 7 : Find the total number of prime factors in the expression (14)11 x (7)2 x (11)3 .
Solution : (14)11 x (7)2 x (11)3 = (2 x 7)11 x (7)2 x (11)3 = (2)11 x (7)11 x (7)2 x (11)3 = (2)11 x (7)13 x (11)3
Therefore, total number of prime factors = 11 + 13 + 3 = 27.
LCM and HCF
• Factors and Multiples: All the numbers that divide a number completely, i.e., without leaving
any remainder, are called factors of that number. For example, 24 is completely divisible by 1,
2, 3, 4, 6, 8, 12, 24. Each of these numbers is called a factor of 24 and 24 is called a multiple
of each of these numbers.
• LCM: The least number which is exactly divisible by each of the given numbers is called the
least common multiple of those numbers. For example, consider the numbers 3, 31 and 62
(2 x 31). The LCM of these numbers would be 2 x 3 x 31 = 186. To find the LCM of the given
numbers, we express each number as a product of prime numbers. The product highest
power of the prime numbers that appear in the prime factorization of any of the numbers
gives us the LCM. For example, consider the numbers 2, 3, 4 (2 x 2), 5, 6 (2 x 3). The LCM of
these numbers is 2 x 2 x 3 x 5 = 60. The highest power of 2 comes from prime factorization of
4, the highest power of 3 comes from prime factorization of 3 and the prime factorization of
6 and the highest power of 5 comes from prime factorization of 5.
• HCF : The largest number that divides two or more numbers is the highest common factor
(HCF) for those numbers. For example, consider the numbers 30 (2 x 3 x 5), 36 (2 x 2 x 3 x 3),
42 (2 x 3 x 7), 45 (3 x 3 x 5). 3 is the largest number that divides each of these numbers, and
hence, is the HCF for these numbers. HCF is also known as the Greatest Common Divisor
(GCD). To find the HCF of two or more numbers, express each number as the product of
prime numbers. The product of the least powers of common prime terms gives us the HCF.
This is the method we illustrated in the above step. Also, for finding the HCF of two numbers,
we can also proceed by long division method. We divide the larger number by the smaller
number (divisor). Now, we divide the divisor by the remainder obtained in the previous
stage. We repeat the same procedure until we get zero as the remainder. At that stage, the
last divisor would be the required HCF. For example, we find the HCF of 30 and 42.
• HCF of co-primes = 1
• For two fractions, HCF = HCF (Numerators) / LCM (Denominators) LCM = LCM (Numerators) /
HCF (Denominators)
Sample Problems
Question 1 :
Two numbers are in the ratio of 5:11. If their HCF is 7, find the numbers.
Solution :
Let the numbers be 5m and 11m. Since 5:11 is already the reduced ratio, 'm' has to be the HCF. So,
the numbers are 5 x 7 = 35 and 11 x 7 = 77.
Question 2 :
Find the length of the plank which can be used to measure exactly the lengths 4 m 50 cm, 9 m 90 cm
and 16 m 20 cm in the least time.
Solution :
Let us first convert each length to cm. So, the lengths are 450 cm, 990 cm and 1620 cm. Now, we
need to find the length of the largest plank that can be used to measure these lengths as the largest
plank will take the least time. For this, we need to take the HCF of 450, 990 and 1620.
450 = 2 x 3 x 3 x 5 x 5 = 2 x 32 x 52
990 = 2 x 3 x 3 x 5 x 11 = 2 x 32 x 5 x 11
1620 = 2 x 2 x 3 x 3 x 3 x 3 x 5 = 22 x 34 x 5
Thus, we need a plank of length 90 cm to measure the given lengths in the least time.
Question 3 :
Find the greatest number which on dividing 70 and 50 leaves remainders 1 and 4 respectively.
Solution :
The required number leaves remainders 1 and 4 on dividing 70 and 50 respectively. This means that
the number exactly divides 69 and 46.
Question 4 :
Find the largest number which divides 64, 136 and 238 to leave the same remainder in each case.
Solution :
To find the required number, we need to find the HCF of (136-64), (238-136) and (238-64), i.e., HCF
(72, 102, 174).
72 = 23 x 32
102 = 2 x 3 x 17
174 = 2 x 3 x 29
Question 5:
Find the least number which when divided by 5,7,9 and 12, leaves the same remainder 3 in each case
Solution:
In these types of questions, we need to find the LCM of the divisors and add the common remainder
(3) to it.
Question 6 :
Find the largest four digit number exactly divisible by 15,21 and 28.
Solution :
Question 7:
The policemen at three different places on a ground blow a whistle after every 42 sec, 60 sec and 78
sec respectively. If they all blow the whistle simultaneously at 9:30:00 hours, then at what time do
they whistle again together?
Solution:
They all will whistle again at the same time after an interval that is equal to the LCM of their
individual whistleblowing cycles.
So, LCM (42, 60, 78) = 2 x 3 x 7 x 10 x 13 = 5460
Therefore, they will blow the whistle again simultaneously after 5460 sec, i.e., after 1 hour 31
minutes, i.e., at 11:01:00 hours.
Question 8 :
Find the least number which when divided by 6,7,8 leaves a remainder 3, but when divided by 9
leaves no remainder
Solution :
Question 9 :
Two numbers are in the ratio 2:3. If the product of their LCM and HCF is 294, find the numbers.
Solution :
Let the common ratio be 'm'. So, the numbers are 2m and 3m.
Now, we know that Product of numbers = Product of LCM and HCF.
=> 2m x 3m = 294
=> m2 = 49
=> m = 7
Question 10 :
A rectangular field of dimension 180m x 105m is to be paved by identical square tiles. Find the size of
each tile and the number of tiles required.
Solution:
We need to find the size of a square tile such that a number of tiles cover the field exactly, leaving no
area unpaved.
For this, we find the HCF of the length and breadth of the field.
Question 11 :
Three rectangular fields having area 60 m2 , 84 m2 and 108 m2 are to be divided into identical
rectangular flower beds, each having length 6 m. Find the breadth of each flower bed.
Solution :
We need to divide each large field into smaller flower beds such that the area of each bed is same.
So, we find the HCF of the larger fields that give us the area of the smaller field.
=> 12 = 6 x Breadth
=> Breadth = 2 m
Question 12:
Find the maximum number of students among whom 182 chocolates and 247 candies can be
distributed such that each student gets the same number of each. Also, find the number of
chocolates and candies each student will get.
Solution:
We need to find the HCF of the number of chocolates and candies available, which would give us the
number of students.
A number is divisible by 2 if its last digit is any of 0, 2, 4, 6, or 8. This is because every number
divisible by 2 can be expressed as an even number, and even numbers always have their last digit as
one of the mentioned digits. For example, 24, 126, and 358 are all divisible by 2 since their last digits
are 4, 6, and 8, respectively.
A number is divisible by 3 if the sum of its digits is divisible by 3. This rule is often referred to as the
divisibility rule for 3.
For example, let's take the number 12321. The sum of its digits is 1 + 2 + 3 + 2 + 1 = 9. Since 9 is
divisible by 3, we can conclude that 12321 is also divisible by 3.
This rule can be applied to any number to determine its divisibility by 3 based on the sum of its
digits.
A number is divisible by 4 if its last two digits form a number that is divisible by 4. This is known as
the divisibility rule for 4.
For example, let's consider the number 1234. The last two digits are 34, and since 34 is not divisible
by 4, we can conclude that 1234 is not divisible by 4.
On the other hand, if we take the number 1232, the last two digits are 32, which is divisible by 4.
Therefore, we can say that 1232 is divisible by 4.
This rule allows us to quickly determine the divisibility of a number by 4 by examining its last two
digits.
A number is divisible by 5 if its last digit is either 0 or 5. This is because numbers divisible by 5 have a
pattern where the last digit repeats in cycles of 5: 0, 5, 0, 5, and so on.
For example, 150 is divisible by 5 since its last digit is 0. Similarly, 735 is also divisible by 5 because its
last digit is 5.
However, if the last digit of a number is not 0 or 5, then the number is not divisible by 5. For instance,
362 is not divisible by 5 because its last digit is 2.
Using the divisibility rule for 5, we can quickly determine if a number is divisible by 5 based on its last
digit.
For example, 114 is divisible by 6 as it is divisible by both 2(last digit is 4) and 3 (1+1+4=6, 6 is
divisible by 3).
Divisibility by 7 : A number is divisible by 7 iff repeatedly doing following steps until a single digit left
leaves the single digit as 0 or 7.
Example, given number is 196. After removing last digit, we get 19. After subtracting 12 (double of
removed digit), we get 7. Since the last left digit is 7, number is multiple of 7.
For example, 1234 is not divisible by 8 as the last three digits 234 is not divisible by 8. But, 1232 is
divisible by 8 as the last three digits 232 is divisible by 8.
Divisibility by 9: A number is divisible by 9 if the sum of its digits is divisible by 9.
Divisibility by 11 : A number is divisible by 11 if the difference between the sum of numbers at even
positions and odd positions is either 0 or a multiple of 11.
NOTE : If 'p' and 'q' are co-primes and we have a number 'n' that is divisible by both 'p' and 'q', 'n'
will be divisible by p x q.
For example, 48 is divisible by both 3 and 8 and also by 3 x 8 = 24.
But, if 'p' and 'q' are NOT co-prime, then the fact that 'n' would be divisible by p x q given that 'n' is
divisible by both 'p' and 'q' is not necessary. For example, 144 is divisible by both 8 and 12 (not co-
prime), but it is not divisible by 8 x 12 = 96.
DIVISION THEOREM
• (xn - an) is divisible by (x - a) for all values of n. For example, for n = 2, x2 - a2 = (x - a) (x + a),
which is divisible by (x - a). Similarly, for n = 3, x3 - a3 = (x - a) (x2 + a2 + xa), which is divisible
by (x - a).
• (xn - an) is divisible by (x + a) for all even values of n. For example, for n = 2, x 2 - a2 = (x - a) (x +
a), which is divisible by (x+a). Similarly, for n = 3, x 3 - a3 = (x - a) (x2 + a2 + xa), which is not
divisible by (x + a).
• (xn + an) is divisible by (x + a) for all odd values of n. For example, for n = 3, x 3 + a3 = (x + a)
(x2 + a2 - xa), which is divisible by (x + a).
Factorial of a number
The factorial of a non-negative integer, denoted by the symbol "!", is the product of all positive
integers less than or equal to that number. It is a common mathematical operation used in
combinatorics, probability, and other areas.
For example:
4! = 4 * 3 * 2 * 1 = 24 7! = 7 * 6 * 5 * 4 * 3 * 2 * 1 = 5040 1! = 1
The factorial function grows very rapidly as the input number increases. The factorial of 0 is defined
to be 1.
Sample Problems
Question 1 : When a number is successively divided by 2, 3, 7, we get 1, 2, 3 as the remainder
respectively. What is the smallest such number ?
Solution : The number is of the form 2a+1, 3b+2, 7c+3. So, we put c=1 and proceed as follows :
Question 3 : Which digits should come in place of * and # such that the number 12386*# is divisible
by both 8 and 5 ?
Solution : Since the given number should be divisible by 5, 0 or 5 must come in place of #. But, a
number ending with 5 is never divisible by 8. So, 0 will replace #.
Now, the number formed by the last three digits is 6*0, which becomes divisible by 8, if * is replaced
by 0 or 4 or 8.
Hence, digits in place of * and # are 0 or 4 or 8 and 0 respectively.
Question 4 : What is the least number that must be subtracted from 9999 to make it exactly divisible
by 19 ?
Solution : On dividing 9999 by 19, we get 5 as remainder. Therefore, number to be subtracted = 5.
Question 5 : What is the least number that must be added to 9999 to make it exactly divisible by 19 ?
Solution : On dividing 9999 by 19, we get 5 as remainder. Therefore, number to be added = 19 - 5 =
14.
Question 6 : A number when divided by 340 gives a remainder 47. What would be the remainder
when the same number is divided by 17 ?
Solution : The number is of the form 340a + 47 = 17 * (20a) + 17 * (2) + 13 = 17 * (20a + 2) + 13.
Therefore, on dividing this number by 17, we would get 13 as the remainder.
Average
The concept of average, also known as the arithmetic mean, is a fundamental statistical
measure used to summarize a set of numerical values. It provides a single representative
value that represents the central tendency of the data. The average is calculated by dividing
the sum of all values by the total number of values.
Formula for Average: Average = (Sum of all values) / (Total number of values)
Example 1: Calculating Average Test Scores Suppose we have the following test scores: 85,
92, 78, 88, and 95. To find the average test score, we can use the formula mentioned earlier.
Average = 250,000/5=50,000
Example 3: Average Age of a Group Suppose we have a group of individuals with ages 20, 25,
18, 22, 27, and 30. Let's find the average age of the group.
Example 4: Average Daily Temperature Consider the daily temperatures recorded over a
week: 25°C, 28°C, 30°C, 27°C, 26°C, 23°C, and 24°C. Let's calculate the average daily
temperature.
Therefore, the average daily temperature over the week is approximately 26.14°C.
Example 1: Average Fuel Efficiency Suppose you want to calculate the average fuel efficiency
of a car for a road trip. You recorded the following distances covered on different days: 250
miles, 200 miles, 180 miles, and 300 miles. The car's fuel tank capacity is 10 gallons.
Solution: Sum of all values = 250 + 200 + 180 + 300 = 930 miles Total number of values = 4
To find the average fuel efficiency in terms of miles per gallon (MPG), divide the average
distance by the fuel consumption: Average MPG = 232.5 miles / 10 gallons = 23.25 MPG
Therefore, the average fuel efficiency of the car for the road trip is approximately 23.25
MPG.
Example 2: Average Time Taken Consider a runner training for a marathon. He recorded the
following times (in minutes) taken to complete different training sessions: 50, 55, 48, 53, and
50 minutes.
Example 3: Average Daily Sales Suppose you own a small business and want to calculate the
average daily sales for a week. The sales amounts (in dollars) for each day are as
follows: 1200,1500, 900,1100, 1400,1000, and $1300.
Solution: Sum of all values = 1200+1500 + 900+1100 + 1400+1000 + 1300=8400 Total
number of values = 7
Average = 8400/7=1200
Therefore, the weighted average test score for the student is 86%.
Problems of Age
Question 1 : A's age after 15 years would be equal to 5 times his age 5 years ago. Find his
age 3 years hence.
Solution : Let A's present age be 'n' years.
According to the question,
n + 15 = 5 (n - 5)
=> n + 15 = 5 n - 25
=> 4n = 40
=> n = 10
=> A's present age = 10 years
Therefore, A's age 3 years hence = 10 + 3 = 13 years
Question 2 : The product of the ages of A and B is 240. If twice the age of B is more than A's
age by 4 years, what was B's age 2 years ago?
Solution : Let A's present age be x years. Then, B's present age = 240 / x years
So, according to question
2 (240 / x ) - x = 4
=> 480 - x2 = 4 x
=> x2 + 4 x – 480 = 0
=> (x + 24) (x - 20) = 0
=> x = 20
=> B's present age = 240 / 20 = 12 years
Thus, B's age 2 years ago = 12 - 2 = 10 years
Question 3 : The present age of a mother is 3 years more than three times the age of her
daughter. Three years hence, mother's age will be 10 years more than twice the age of the
daughter. Find the present age of the mother.
Solution : Let the daughter's present age be 'n' years.
=> Mother's present age = (3n + 3) years
So, according to the question
(3n + 3 + 3) = 2 (n + 3) + 10
=> 3n + 6 = 2n + 16
=> n = 10
Hence, mother's present age = (3n + 3) = ((3 x 10) + 3) years = 33 years
Question 4 : The ratio of present ages of A and B is 6 : 7. Five years hence, this ratio would
become 7 : 8. Find the present age of A and B.
Solution : Let the common ratio be 'n'.
=> A's present age = 6 n years
=> B's present age = 7 n years
So, according to the question
(6 n + 5) / (7 n + 5) = 7 / 8
=> 48 n + 40 = 49 n + 35
=> n = 5
Thus, A's present age = 6 n = 30 years and B's present age = 7 n = 35 years
Logarithms
Concept and Properties of Logarithms:
Logarithms are mathematical functions that express the relationship between exponents
and bases. They are widely used in various fields, including mathematics, science,
engineering, and finance. Logarithms provide a convenient way to solve exponential
equations and perform calculations involving large numbers.
Definition: In simple terms, the logarithm of a number 'x' to a given base 'b' is the exponent
to which the base 'b' must be raised to obtain the number 'x'. It is represented as log base b
of x or simply logb x.
1. Logarithm as an Exponent: The logarithm of a number 'x' to the base 'b' is the
exponent 'y' such that bx = x.
3. Logarithm of the Base: The logarithm of the base 'b' to itself is always 1. In other
words, logb b = 1.
4. Product Rule: The logarithm of the product of two numbers is equal to the sum of
their logarithms. It is expressed as log b x * logb y = logb (xy).
5. Quotient Rule: The logarithm of the division of two numbers is equal to the
difference of their logarithms. It is expressed as log b x / logb y = logb (x/y).
6. Power Rule: The logarithm of a number raised to a power is equal to the product of
the power and the logarithm of the number. It is expressed as log b (x^y) = y * logb x.
7. Change of Base Formula: If we want to calculate the logarithm of a number 'x' to a
different base 'c', we can use the change of base formula. It is expressed as log c (x) =
logb (x) / logb (c).
Example 1: Solve an Exponential Equation Find the value of 'x' in the equation 3 x = 27.
Solution: We can rewrite the equation using logarithms: x = log 3 (27). Using the base-3
logarithm, we find x = 3, as 3 3 = 27.
Example 3: Change of Base Calculation. Calculate log 5 (125) using the change of base
formula.
Solution: Using the change of base formula, we have log 5 (125) = log10 (125) / log10 (5).
Evaluating the logarithms, we get log5 (125) = 3 / 0.69897 (approximately). Simplifying
further, we find log5(125) ≈ 4.177.
Percentages
Concept and Properties of Percentages:
Percentages are a common way to express proportions or ratios as a fraction of 100. They
are widely used in everyday life, business, finance, and statistics. Understanding the concept
and properties of percentages allows for easier comparison, calculation, and interpretation
of values.
• To convert a fraction to percentage, we multiply by 100 and add the "%" sign. For
example, to express 1 / 5 in percentage, we simply multiply by 100, (1 / 5) x 100 = 20
%
• x % of y and y % of x are same. For example, 10 % of 100 and 100 % of 10 are same.
Note - If there is % decrease instead of % increase, then we take (-) negative sign.
Solution: Percentage Increase = [(New Value - Old Value) / Old Value] * 100 = [(100 - 80) /
80] * 100 = (20 / 80) * 100 = 0.25 * 100 = 25%
The percentage increase is 25%.
Example 2: Finding a Percentage Out of a class of 40 students, 30 students passed the exam.
What percentage of students passed?
Solution: Percentage = (Number of Students Passed / Total Number of Students) * 100 = (30
/ 40) * 100 = 0.75 * 100 = 75%
Example 3: Percentage Discount An item originally priced at $200 is on sale with a 20%
discount. What is the discounted price?
Solution: Discounted Price = Original Price - (Discount Percentage * Original Price)
= 200−(0.20∗200) = 200−40 = $160
Example 4: Finding a Number Given a Percentage 25% of a number is 50. Find the number.
Solution: Let's assume the number is 'x'. 25% of x = 50 (25/100) * x = 50 0.25 * x = 50 x = 50 /
0.25 x = 200
Question 1 : A defect finding machine rejects 0.085% of all the cricket bats. Find the number
of bats manufactured on a particular day if it is given that on that day, the machine rejected
only 34 bats.
Solution : Let the total number of bats on that day be n.
=> 0.085 % of n = 34
=> (0.085 / 100) x n = 34
=> n = 34 x (100 / 0.085)
=> n = 40,000
Therefore, total number of bats manufactured on the day = 40,000
Question 2 : 25 % of a number is 8 less than one third of that number. Find the number.
Solution : Let the number be n.
=> (n / 3) - 25 % of n = 8
=> (n / 3) - (n / 4) = 8
=> n / 12 = 8
=> n = 96
Thus, 96 is the required number.
Question 3 : Difference of two numbers 'x' and 'y' (x > y) is 100. Also, 10 % of 'x' is equal to
15 % of 'y'. Find the numbers.
Solution : We are given that x - y = 100 and 10 % of x = 15 % of y
=> x - y = 100 and (10 / 100) x = (15 / 100) y
=> x - y = 100 and 10 x = 15 y
=> x - y = 100 and 2 x = 3 y
=> x - y = 100 and x = 1.5 y
=> 1.5 y - y = 100
=> 0.5 y = 100
=> y = 200
=> x = 1.5 y = 300
Thus, the required numbers are 300 and 200.
Question 4 : In a gaming event, 75 % of the registered participants actually turned up. Out of
those, 2 % were declared unfit for participation. The winner defeated 9261 participants
which are 75 % of the total valid participation. Find the number of registered participants.
Solution : Let the number of registered participants be n.
Number of participants who actually turned up = 75 % of n
Number of valid participations = 98 % of (75 % of n) [because 2% were invalid]
Number of participants defeated by the winner = 75 % of 98 % of (75 % of n) = 9261
=> 0.75 x 0.98 x 0.75 x n = 9261
=> 0.55125 x n = 9261
=> n = 16800
Therefore, number of registered participants = 16800
Question 6 : A geek gave 40% of his retirement money to his wife. He also gave 20 % of the
remaining amount to each of his 3 sons. 50 % of the amount now left was spent on
miscellaneous items and the remaining amount of Rs. 1,20,000 was deposited in the bank.
How much money did the geek got as the retirement money?
Solution : Let the retirement money be Rs. 100 n
=> Money given to wife = 40 % of 100 n = 40 n, Balance = 60 n
=> Money given to 3 sons = 3 x (20 % of 60 n) = 3 x 12 n = 36 n, Balance = 24 n
=> Money spent on miscellaneous items = 50 % of 24 n = 12 n, Balance = 12 n
Now, this remaining 12 n is the money deposited in the bank, i.e., Rs. 1,20,000
=> 12 n = 1,20,000
=> n = 10,000
Therefore, geek's retirement money = 100 n = Rs. 10,00,000
Question 7 : A broker charges commission of 5% on all orders upto Rs.10,000 and 4% on all
orders exceeding Rs. 10,000. He remits Rs. 31,100 to his client after deducting his
commission. Find the order amount.
Solution : Let the order amount be Rs. n
=> Commission charged = 5 % of Rs. 10,000 + 4 % of (Rs. n - 10,000) = Rs. 500 + 0.04 n - 400
=> Commission charged = Rs. 100 + 0.04 n
Now, amount remitted = Rs. n - (100 + 0.04 n) = 31,100
=> 0.96 n - 100 = 31,100
=> 0.96 n = 31200
=> n = 32500
Therefore, order amount = Rs. 32,500
Question 8 : A seller marked up the price of an article by 20 % and then gave a discount of
20 %. Find what percent did he lose in the transaction.
Solution : We know that an increase by n % and a successive decrease by n % is equal to an
equivalent decrease of (n/10) 2 %.
=> Net decrease or loss of the seller = (20/10) 2 = 4 %
Question 11 : The population of a town is 1,60,000 in the current year. If it increases at the
rate of 5% per annum, what will be the population 3 years from now ?
Solution : We know that if the population is currently P and it increases by R % every year,
then, population after 'n' years = P x [1 + (R / 100)] n
=> Population after 3 years = 1,60,000 x [1 + (5 / 100)] 3
=> Population after 3 years = 1,60,000 x (1.05) 3
=> Population after 3 years = 1,60,000 x 1.157625 = 185220
Question 12 : The value of a car is Rs 1,60,000 in the current year. If it depreciates at the rate
of 5% per annum, what will be the value of the car 3 years from now ?
Solution : We know that if the value is currently P and it depreciates by R % every year, then,
value after 'n' years = P x [1 - (R / 100)] n
=> Value after 3 years = 1,60,000 x [1 - (5 / 100)] 3
=> Value after 3 years = 1,60,000 x (0.95) 3
=> Value after 3 years = 1,60,000 x 0.857375 = Rs. 1,37,180
Question 13 : How much sugar (in KG) must be added to 50 KG of a 2 % sugar solution so as
to make the concentration 10 % ?
Solution : Sugar in initial solution = 2 % of 50 KG = 1 KG
Let the sugar added be n KG.
=> (1 + n) / (50 + n) = 10 / 100
=> n = 40 / 9
Therefore, 40 / 9 KG of sugar should be added.
8. Cost Price (after discount) Cost Price = [100 / (100 – Loss%)] x Selling Price
Q1: A person buys a pen from a wholesaler at Rs. 10 for 20 pens. He sells those pens at Rs.
10 for 15 pens. Find his profit or loss percent.
Solution:
CP for each pen = 10 / 20 = Rs. 0.50 SP for each pen = 10 / 15 = Rs. 2 / 3 Profit = SP – CP = Rs.
(2 / 3) – 0.50 = Rs. 1 / 6 Therefore, profit percent = [ (1/6) / (0.50) ] x 100 = 33.334%
Q2: A dealer incurs a loss of 5 % if he sells an article for Rs. 1805. What price must he sell
the article so as to gain 5 % on that article?
Solution:
Let the cost price of the article be Rs. C => SP = CP – Loss => 1805 = C – 0.05 C => 0.95 C =
1805 => C = 1900 Therefore, to gain 5 %, SP = 1900 + (0.05 x 1900) = 1900 + 95 = Rs. 1995
Q3: If the cost price of an article is 67 % of the selling price, what is the profit percent?
Solution:
Let the selling price of the article be Rs. S => Cost price of the article = 67 % of S = 0.67 S =>
Profit = SP – CP = 0.33 S Therefore, profit percent = (0.33 S / 0.67 S) x 100 = 49.25 %
Q4: A shopkeeper purchased two varieties of rice, 80 KG at Rs. 13.50 per KG and 120 KG at
Rs. 16 per KG. The shopkeeper being greedy, mixed the two varieties of rice and sold the
mixture at a gain of 16 %. Find the per KG selling price of the mixture.
Solution:
We are given that the shopkeeper bought 80 Kg at Rs. 13.50 per KG and 120 KG at Rs. 16 per
KG. => Total cost price = (80 x 13.50) + (120 x 16) = 1080 + 1920 = Rs. 3000 and total rice = 80
+ 120 = 200 KG Now, total selling price = Total cost price + 16 % of total cost price => Total
selling price = 3000 + (0.16 x 3000) = Rs. 3480 Thus, selling price per KG = 3480 / 200 = Rs.
17.40
=> (m – 13.50) / (16 – m) = 120 / 80 => m = 15, where ‘m’ is the per KG cost price of the
mixture Therefore, per KG selling price of the mixture = Rs. 15 + 16% of 15 = Rs. 17.40
Q5: A seller claims to sell at cost price but gives 750 gm for each KG. Find his gain percent.
Solution :
Profit percent = [ (True Value – Given Value) / Given Value ] x 100 % Here, True Value = 1 KG =
1000 gm Given Value = 750 gm Therefore, profit percent = [ (1000 – 750) / 750 ] x 100 = (250
/ 750) x 100 = 33.334 %
Q6: A man sold two watches at the same price, one at a 10 % profit and the other at a 10
% loss. Find his overall gain or loss percentage.
Solution:
We know that if two articles are sold at the same selling price, one at a gain of A% and one
at the loss of A%, then the seller always incurs a loss of (A / 10)2. => Loss percent = (10 / 10)2
=1%
Long Method:
Let the selling price of each watch be Rs. 99 S => Total SP = Rs. 198 S CP of first watch = SP –
Profit = Rs. 99 S- 10 % of CP = Rs. 90 S CP of second watch = SP + Loss = Rs. 99 S + 10 % of CP
= Rs. 110 S => Total CP = Rs. 90 S + 110 S = Rs. 200 S => Loss = Total CP – Total SP = 200 – 198
= Rs. 2 S Therefore, loss percent = (Loss / CP) x 100 = (2 S / 200 S) x 100 % = 1 %
Let the marked price of the shirt be Rs. 1000 => Price after first discount = Rs. 1000 – 20 % of
Rs. 1000 = Rs. 1000 – 200 = Rs. 800 => Price after second discount = Rs. 800 – 10 % of Rs. 800
= Rs. 800 – 80 = Rs. 720 => Price after cash discount = Rs. 720 – 5 % of Rs. 720 = Rs. 720 – 36
= Rs. 684 Therefore, total discount = Rs. 1000 – 684 = Rs. 316 => Overall discount percent =
(316 / 1000) x 100 = 31.60 %
Q8: A dealer wants to mark the price of an article such that by offering a 5 % discount, he
is able to get 33 % profit. Find the percent of CP above which the article should be
marked.
Solution:
Let the cost price of the article be Rs. 100 => Selling price of the article = Rs. 100 + 33% of CP
= Rs. 133 Let the marked price be Rs. M => Selling price = Marked Price – Discount => 133 =
M – 0.05 M => 133 = 0.95 M => M = 140 => M – CP = 140 – 100 = 40 Therefore, percent of CP
above which the article should be marked = (40 / 100) x 100 = 40 %
Q.1 Sathish bought a second-hand bike and spent Rs.3000 on its repair and spare
parts. Then he sold it to Rajesh at a profit of 15 % and then Rajesh sold it to Suresh at a
loss of 10 %. Suresh finally sold it for Rs.22770 at a profit of 10%. How much amount did
Sathish initially pay for the bike?
A.Rs. 21000
B.Rs. 17000
C.Rs. 23000
D.Rs. 19000
E. None of these
Answer: B
Let Sathish pay for the bike be Rs. x,
(x + 3000)*(115/100)*(90/100)*(110/100) =
22770
(x + 3000) = (22770*100*100*100)/(115*90*110)
(x + 3000) = 20000
x = 17000
The initial amount paid by Sathish = Rs. 17000
Q.2 A shopkeeper marks the price of the article as is 20% above its cost price and also
offers a 10% discount. Find the profit percentage?
A.10%
B. 8%
C.12%
D.14%
E. None of these
Answer: B
CP = x
SP = x * 120/100 * 90/100
= 1.08x
Profit = 1.08x – x
= 0.08x
Profit percentage = 0.08x/x * 100
= 8%
Q.3 The shopkeeper sold article A at a 20% loss and the cost price of article A is Rs.4500.
With that amount, he bought article B and sold it at a profit of 30%. What is the overall
profit or loss in the whole transaction?
A. Rs.120
B. Rs.150
C. Rs.180
D. Rs.200
E. None of these
Answer: C
Q.4 A shopkeeper offers two successive discounts of 20% and 15% respectively for the
laptop while he gets 20% profit. If the selling price of the laptop is Rs.34000, what is the
discount amount?
A.Rs.12000
B.Rs.14000
C.Rs.16000
D.Rs.18000
E. None of these
Answer: C
SP = 34000
MP * 80/100 * 85/100 = 34000
MP = 50000
Discount = 50000 – 34000 = 16000
Q.5 A shopkeeper bought two articles A and B such that he had a Profit of 15% and 20% on
selling articles A and B respectively. The selling price of articles A and B is Rs.230 and
Rs.420 respectively. At what price must that items be
sold together in order to gain 30%?
A. Rs.686
B. Rs.695
C. Rs.700
D. Rs.740
E. None of these
Answer: E
SP of A = 230
Profit% on A = 15%
CP of A = 230 * 100/115 = 200
SP of B = 420
Profit% on B = 20%
CP of B = 420 * (100/120)= 350
Total CP = 200 + 350 = 550
Now, to earn a total profit of 30% on both item
together, selling price = 130% of 550
= Rs.715
Q.6 What is the value of y, if an article is sold after allowing two successive discounts of y%
and 5% and per cent profit earned after selling the article is y%. The cost price and Marked
price of the article are Rs.2850 and Rs.5000 Respectively.
A.15
B.20
C.25
D.30
E.35
Answer: C
CP = 2850 MP = 5000
Profit = y% Successive
discounts = y%, 5%
((100 – y)/100) * ((100 -5)/100) * 5000
= ((100+y)/100) * 2850 [(100 – y)/100] * 95/100 * 5000
= (100 + y)/100 * 2850
(100 – y) = (100+y) * 3/5
500 – 5y = 300 + 3y
8y = 200
y = 25
Q.7 The shopkeeper marked 20% above the cost price of the article and also he gives 15%
of the discount to his favourite customer. The customer sold that article to his friend in the
amount of Rs.5896. If the cost price of the article is Rs.4800, then what is the profit gained
by the customer?
A.Rs.800
B.Rs.2000
C.Rs.1000
D.Rs.1500
E. Cannot be determine
Answer: C
MP of the article=4800 * 120/100=Rs.5760
SP of the article =5760 * 85/100=Rs.4896
CP of article for Customer=Rs.4896
SP of the customer=Rs.5896 Profit=5896 – 4896=1000
Q.8 By selling a laptop for Rs.12000, a shopkeeper gains 20%. If the profit is reduced to
15%, then find the selling price of the laptop?
A.Rs.11500
B.Rs.13480
C.Rs.14560
D.Rs.23490
E. None of these
Answer: A
CP = SP * 100/(100 + P%)
= 12000 * 100/(100 + 20)
= Rs.10000
SP of laptop for 15% profit = CP X (100 +
P%)/100
= 10000 * (100 + 15)/100
= Rs.11500
Q.9 Praveen is a computer shop owner. He purchases the components of a desktop for Rs.
6200 and then he spends a further Rs. 2400 on labour charge to assemble the desktop. If
he sells 18 desktops for Rs. 175000, then what is the approximate profit percentage?
A.13%
B.11.5%
C.12.2%
D.14.35%
E. 8.9%
Answer: A
Q.10 A man sold an article for Rs.7600 and incurred a loss. Had he sold the article for
Rs.8350, his gain would have been equal to half of the amount of loss that he incurred. At
what price should he sell the article to have a 20% profit?
A. Rs.8560
B. Rs.7680
C. Rs.11240
D. Rs.9720
E. None of these
Answer: D
Q.11 The selling price of 8 articles is equal to the cost price of 10 articles. What is the
percentage profit in selling the articles?
A. 15%
B. 12%
C. 20%
D. 25%
E. 28%
Answer: D
A. Rs.3
B. Rs.60
C. Rs.57
D. Rs.51
E. Rs.253
Answer: C
CP = 50
MP = 50 * 120/100 = 60
Discount = 60 * 5/100 = 3
Total discount = 3 * 19 = 57
Q.13 Ratio of the cost price to the marked price of the article is 4: 5. A shopkeeper offers a
discount of 15% and the marked price of the article is Rs.720. What is the profit
percentage of the article?
A. 6.25%
B. 4.75%
C. 5.5%
D. 3.40%
E. None of these
Answer: A
CP = 4/5 * 720 = Rs.576
SP = 720 * 85/100 = Rs.612
Profit percentage = (612 – 576)/576 * 100
= 6.25%
Q.14 A fruit seller sold fruit at a loss of 5%. Had he sold it for Rs.20 more, he would have
earned a profit of 5%. Find the cost of the fruit.
A. Rs.100
B. Rs.200
C. Rs.150
D. Rs.250
E. None of these
Answer: B
Q.15 A shopkeeper bought 60kg of apple at the rate of Rs.90 per kg. He sold 45% of the
total quantity at the rate of Rs.100 per kg. At what price per kg (approximate value) should
he sell the remaining quantity to make a 25% overall profit?
A. Rs.123
B. Rs.139
C. Rs.137
D. Rs.143
E. Rs.113
Answer: A
CP=90*60
=Rs.5400
45% of 60kg
=60*45/100
=27kg
SP of 27kg
=27X100
=Rs.2700
25% of profit
=5400X125/100
=Rs.6750
Amount=6750-2700=4050
Remaining apple=60-27=33kg
Required amount=4050/33
=Rs.122.72
Q.16 If the selling price of the mobile is Rs.6000 and the ratio of the marked to cost price
of the mobile is 5: 3. If the shopkeeper offers a discount of 20% on the marked price of the
mobile, then find the profit percentage of the mobile?
A. 28.28%
B. 44.44%
C. 33.33%
D. 48.48%
E. 52.52%
Answer: C
SP of mobile = Rs.6000
MP of mobile = 5x
CP of mobile = 3x
5x * 80/100 = 6000
x = 1500
CP of mobile = 3 * 1500 = Rs.4500
Profit percentage = (6000 – 4500)/4500 * 100 =
33.33%
Q.17 Ratio of the cost price of the mobile and the selling price of the laptop is 1:1 and the
shopkeeper sold the mobile at a 10% loss and the laptop at a 20% profit. If the selling price
of a mobile and laptop together is Rs.19000, what is the approximate total profit or loss?
A. No gain no loss
B. Rs.555 loss
C. Rs.667 profit
D. Rs.480 loss
E. Rs.499 profit
Answer: C
CP of mobile = m
CP of laptop = l
SP of mobile = x
SP of laptop = y
x+ y = 19000
m=y
x = 90/100 * m
x = 90/100 * y
((90/100) * y) + y = 19000
190y/100 = 19000
y = 10000
x = 9000
m = 9000 * 100/90 = 10000
y = 120/100 * l
10000 = 120/100 * l
l = 1000000 / 120 = 50000/6
Total CP = 10000 + 50000/6 = Rs.18333.33
Profit = 19000 – 18333.33 = 666.667
Q.18 The marked price of a computer is Rs.15000. If a shopkeeper earned a profit of 16
(2/3)% after giving a phone for purchasing the computer and the cost price is 12000, find
the percentage of discount which is equal to the price of the phone.
A. 8.66%
B. 7.66%
C. 6.66%
D. 6.33%
E. 9.66%
Answer: C
CP = 12000
MP = 15000
Profit = 12000 * 50/300 = 2000
Price of phone = 15000 – (12000 + 2000) =
1000
Discount = 1000
% Discount = (1000/15000) * 100 = 6.66)%
Q.19 Anmol sold an article marked at 20000 after giving four successive discounts of 10%,
15%, 20% and 25%. Instead, if he sold it at a 30% discount, find his net gain or loss in the
Selling price.
A. 4820 loss
B. 2410 gain
C. 4820 gain
D. 2410 loss
E. None of these
Answer: C
1st case:
SP = 20000 * 90/100 * 85/100 *80/100 *75/100 = 9180
2nd case:
SP = 20000 *70/100 = 14000
Therefore, he would have gained = 4820
Q.20 Rajesh bought 23kg of rice at 30 rupees per kg and 32 kg of rice at 35 rupees per kg.
Now he sold the entire rice at 42 rupees per kg. Find the amount of loss/profit made by
Rajesh.
A. Rs.320 profit
B. Rs.410 profit
C. Rs.350 profit
D. Rs.500 profit
E. None of these
Answer: D
• Ratio of two quantities 'a' and 'b' having same units is simply a / b and is usually
written as a:b
• Mean proportional is the geometric mean. For example, the mean proportional of 'a'
and 'b' is square root of (a x b)
• If we say that 'a' is directly proportional to 'b', it means that a = k x b, where 'k' is the
constant of proportionality
Partnership
• When more than one person is involved in a business, it is said to be running in
partnership.
• The gains/losses from the business are divided in the ratio of their inputs, where
input is calculated as the product of the amount of investment and time period of
investment. If A and B invest Rs. V1 and Rs. V2 in business for a time period of T1 and
T2 respectively, then the profit/loss from the business is divided in the ratio (V1 x T1)
: (V2 x T2) The formula gets a summation if some amount is invested for a part of the
total time period and some other amount is invested for the remaining time period.
• For the same period of investment, the profit/loss from the business is divided in the
ratio of the value of investments, i.e., V1 : V2
Sample Problems
Solution : Here, we make the common term 'b' equal in both ratios.
Thus, a : b : c = 35 : 63 : 36
(0.23×0.24)
= 0.234946802
981 / 9 = 109
=> k = 2
Question 5 : A mixture contains sugar solution and colored water in the ratio 4 : 3. If 10 liters
of colored water is added to the mixture, the ratio becomes 4 : 5. Find the initial quantity of
sugar solution in the given mixture.
Final ratio = 4 k : 3 k + 10 = 4 : 5
=> k = 5
Question 6 : Two friends A and B started a business with initial capital contribution of Rs. 1
lac and Rs. 2 lacs. At the end of the year, the business made a profit of Rs. 30,000. Find the
share of each in the profit.
Solution : We know that if the time period of investment is same, profit/loss is divided in the
ratio of value of investment.
=> Ratio of value of investment of A and B = 1,00,000 : 2,00,000 = 1 : 2
Question 7 : Three friends A, B and C started a business, each investing Rs. 10,000. After 5
months A withdrew Rs. 3000, B withdrew Rs. 2000 and C invested Rs. 3000 more. At the end
of the year, a total profit of Rs. 34,600 was recorded. Find the share of each.
Solution: We know that if the period of investment is not uniform, the gains/losses from the
business are divided in the ratio of their inputs, where input is calculated as the product of
the amount of investment and time period of investment.
So, input = value of investment x period of investment, and here, the period of investment
would be broken into parts as the investment is not uniform throughout the time period.
Question 8 : A invested Rs. 70,000 in a business. After few months, B joined him with Rs.
60,000. At the end of the year, the total profit was divided between them in ratio 2 : 1. After
how many months did B join?
=> 12 - n = 7
=> n = 5
• If a pipe can fill a tank in 'n' hours, then in 1 hour, it will fill '1 / n' parts. For example,
if a pipe takes 6 hours to fill a tank completely, say of 12 liters, then in 1 hour, it will
fill 1 / 6 th of the tank, i.e., 2 liters.
• If a pipe can empty a tank in 'n' hours, then in 1 hour, it will empty '1 / n' parts. For
example, if a pipe takes 6 hours to empty a tank completely, say of 18 liters, then in 1
hour, it will empty 1 / 6 th of the tank, i.e., 3 liters.
• If we have a number of pipes such that some fill the tank and some empty it, and we
open all of them together, then in one hour, part of the tank filled/emptied = ∑ (1 /
mi) - ∑ (1 / nj), where 'mi' is the time taken by inlet pipe 'i' to fill the tank completely if
only it were open and 'n j' is the time taken by outlet pipe 'j' to empty the tank
completely if only it were open. If the sign of this equation is positive, the tank would
be filled and if the sign is negative, the tank would be emptied.
Sample Problems
Question 1 : Two pipes A and B can fill a tank separately in 12 and 16 hours respectively. If
both of them are opened together when the tank is initially empty, how much time will it
take to completely fill the tank?
=> Part of tank filled by pipe A and pipe B in one hour working together = (1 / 12) + (1 / 16) =
7 / 48
Therefore, time is taken to completely fill the tank if both A and B work together = 48 / 7
hours
Another Method
Question 2 : Three pipes A, B and C are connected to a tank. Out of the three, A and B are
the inlet pipes and C is the outlet pipe. If opened separately, A fills the tank in 10 hours, B
fills the tank in 12 hours and C empties the tank in 30 hours. If all three are opened
simultaneously, how much time does it take to fill/empty the tank?
Solution : Part of tank filled by pipe A in one hour working alone = 1 / 10
=> Part of tank filled by pipes A,B and C in one hour working together = (1 / 10) + (1 / 12) - (1
/ 30) = 3 / 20
Therefore, time is taken to completely fill the tank if both A and B work together = 20 / 3
hours = 6 hours 40 minutes
Another Method
Let the capacity of tank be LCM (10, 12, 30) = 60 units
=> Efficiency of pipe C = - 60 / 30 = - 2 units / hour (Here, '-' represents outlet pipe)
=> Combined efficiency of pipes A, B and C = 6 + 5 - 2 = 9 units / hour
Question 3 : Three pipes A, B and C are connected to a tank. Out of the three, A is the inlet
pipe and B and C are the outlet pipes. If opened separately, A fills the tank in 10 hours, B
empties the tank in 12 hours and C empties the tank in 30 hours. If all three are opened
simultaneously, how much time does it take to fill/empty the tank?
=> Part of tank filled by pipes A, B and C in one hour working together = (1 / 10) - (1 / 12) - (1
/ 30) = -1 / 60
Therefore, time is taken to completely empty the tank if all pipes are opened simultaneously
= 1 / 60 hours = 60 hours
Another Method
=> Efficiency of pipe C = - 60 / 30 = - 2 units / hour (Here, '-' represents outlet pipe)
=> Combined efficiency of pipes A, B and C = 6 - 5 - 2 = - 1 units / hour (Here, '-' represents
outlet pipe)
Therefore, time taken to completely empty the tank = 60 / (1) = 60 hours
Question 4 : A cistern has two pipes. Both working together can fill the cistern in 12 minutes.
The first pipe is 10 minutes faster than the second pipe. How much time would it take to fill
the cistern if the only second pipe is used?
Solution : Let the time taken by first pipe working alone be 't' minutes.
=> Part of tank filled by pipe A and B in one hour working together = (1 / t) + (1 / t+10) = (2t
+ 10) / [t x (t + 10)]
But we are given that it takes 12 minutes to completely fill the cistern if both pipes are
working together.
=> (t - 20) (t + 6) = 0
Another Method
Let the time taken by the first pipe working alone be 't' minutes.
But we are given that it takes 12 minutes to completely fill the cistern if both pipes are
working together.
=> (t - 20) (t + 6) = 0
Question 5 : Three pipes A, B and C are connected to a tank. Out of the three, A and B are
the inlet pipes and C is the outlet pipe. If opened separately, A fills the tank in 10 hours and
B fills the tank in 30 hours. If all three are opened simultaneously, it takes 3 0 minutes extra
than if only A and B are opened. How much time does it take to empty the tank if only C is
opened?
Therefore, time is taken to completely fill the tank if only A and B are opened = 30 / 4 = 7
hours 30 minutes
=> Time is taken to completely fill the tank if all pipes are opened = 7 hours 30 minutes + 30
minutes = 8 hours
Now, the efficiency of pipe C = Combined efficiency of all three pipes - Combined efficiency
of pipes A and B
Thus, time taken to empty the tank if only C is opened = 30 / 0.25 = 120 hours
Question 6 : Time required by two pipes A and B working separately to fill a tank is 36
seconds and 45 seconds respectively. Another pipe C can empty the tank in 30 seconds.
Initially, A and B are opened and after 7 seconds, C is also opened. In how much more time
the tank would be completely filled ?
Solution : Let the capacity of the tank be LCM (36, 45, 30) = 180 units
Question 7 : Two pipes A and B can fill a tank in 20 hours and 30 hours respectively. If both
the pipes are opened simultaneously, find after how much time should pipe B be closed so
that the tank is full in 18 hours?
Solution : Let the capacity of the tank be LCM (20, 30) = 60 units
=> 5n + 3 x (18 - n) = 60
=> 2n + 54 = 60
=> 2n = 6
=> n = 3
• In case we have more than one type of workers, then the formula modifies
to ∑(Mi Ei) D1 H1 / W1 = ∑(Mj Ej ) D2 H2 / W2, where 'i' and 'j' may vary as per the
number of workers.
• If a person A is 'n' times more efficient than person B, then Ratio of work done by A
and B in one day (Ratio of efficiencies) = n : 1 Ratio of time taken by A and B = 1 : n
• If a group of people are given salary for a job they do together, their individual
salaries are in the ratio of their individual efficiencies if they work for same number
of days. Otherwise, salaries are divided in the ratio of units of work done.
In this topic, the way of attempting the questions is the deciding factor for getting accurate
answer in less time. We shall try to cover all the types of questions asked in this topic with
detailed explanation of way of attempting them.
Sample Problems
Question 1 :
To complete a work, a person A takes 10 days and another person B takes 15 days. If they
work together, in how much time will they complete the work ?
Solution :
Method 1 :
Therefore, working together, they can complete the total work in 6 days.
=> In one day, A and B working together can finish of 5 units of work, out of the given 30
units.
Question 2:
Two friends A and B working together can complete an assignment in 4 days. If A can do the
assignment alone in 12 days, in how many days can B alone do the assignment?
Solution :
Question 3 :
Three people A, B and C are working in a factory. A and B working together can finish a task
in 18 days whereas B and C working together can do the same task in 24 days and A and C
working together can do it in 36 days. In how many days will A, B and C finish the task
working together and working separately?
Solution :
Also, to find the individual times, we need to find individual efficiencies. For that, we
subtract the combined efficiency of any two from the combined efficiency of all three.
Therefore, the time required by A to complete the task alone = 72/1.5 = 48 days
The time required by B to complete the task alone = 72/2.5 = 28.8 days
Question 4:
Two friends A and B are employed to do a piece of work in 18 days. If A is twice as efficient
as B, find the time taken by each friend to do the work alone.
Solution :
=> Total work = No. of Days x Efficiency = 18 days x 3 units / day = 54 units
Therefore, the time required by A to complete the work alone = 54/2 = 27 days
The time required by B to complete the work alone = 54/1 = 54 days
Question 5:
Two workers A and B are employed to do cleanup work. A can clean the whole area in 800
days. He works for 100 days and leaves the work. B working alone finishes the remaining
work in 350 days. If A and B would have worked for the whole time, how much time would it
have taken to complete the work?
Solution :
So, time is taken to complete the work if both A and B would have worked for the whole
time = 800 / 3 = 266.667 days
Question 6:
Three workers A, B and C are given a job to paint a room. At the end of each day, they are
given Rs. 800 collectively as wages. If A worked alone, the work would be completed in 6
days. If B worked alone, the work would be completed in 8 days. If C worked alone, the work
would be completed in 24 days. Find their individual daily wages.
Solution :
Let the total work be LCM (6, 8, 24) = 24 units.
Also, it is given that they get Rs. 800 collectively at the end of each day.
Question 7:
A person A can do a piece of work in 9 days, whereas another person B can do the same
piece of work in 12 days. Because of the busy schedule, they decide to work one day
alternately. If B is the first one to start, find the time required for the work to be completed.
Consider that if a part of the day is used, the whole day is to be counted.
Solution :
Let the total work be LCM (9, 12) = 36 units
=> In 5 such cycles of alternate working, i.e., 10 days, they would have completed 35 units of
work.
Question 8 :
45 men can dig a canal in 16 days. Six days after they started working, 30 more men joined
them. In how many more days will the remaining work be completed?
Solution :
Let the efficiency of each man be 1 unit / day.
Alternate Method
Here, we can use the formula for comparison of work and efficiency
M1 D1H1E1 / W 1= M2D2H2E2/ W 2
Question 9 :
2 Men and 3 Women working together can finish a job in 10 days. It takes 8 days to finish the
same job if 3 Men and 2 Women are employed. If only 2 Men and 1 Woman are employed,
find the time they would take to complete the job.
Solution :
Here, we need to use the summation formula for comparison of work and efficiency
Here, ∑(MiEi) = 2M + 3W, where M is the efficiency of each Man and W is the efficiency of
each Woman
∑(MjEj) = 3M + 2W
D1= 10
D2= 8
Also, H1 = H2 and W1 = W2
Now, we again apply the summation formula with LHS being any of the given set of values
and RHS being the set of values corresponding to 2 Men and 1 Woman.
Therefore, (2M + 3W) x 10 = (2M + 1W) x D, where D is the number of days required to
complete the work if 2 Men and 1 woman are employed.
=> 20k x 10 = 16k x D
To complete a job, A alone takes 2 more days than A and B together. B alone takes 18 more
days than A and B together. Find the time taken if they work together.
Solution :
Total work done by both A and B in one day alone = 1/(n+2) + 1/(n+18)
But, total work done in one day if both A and B work together = 1/n
Therefore, 1/(n+2) + 1/(n+18) = 1/n
Therefore, time taken to complete the job if both A and B work together = 6 days
Short Method
(NOTE : This short cut is applicable if only two people are working on a job)
So, n2= 2 x 18 = 36
=> n = 6.
Therefore, time taken to complete the job if both A and B work together = 6 days
Time, Speed and Distance
Time, Speed and Distance (popularly known as TSD) is an important topic for written round
of placements for any company.
Distance = Speed x Time
If a person covers a certain same distance multiple times with different speeds, then Average
speed = n / ∑ (1/si), where n is the number of times the distance is covered and s i are the
respective speeds of covering the distance. For example, if a person travels a distance of 10
km three times at the speeds of 4 km/h, 5 km/h and 6 km/h, then the average speed would
be 3 / [ (1 / 4) + (1 / 5) + (1 / 6) ] = 3 / (37 / 60) = 180 / 37 ≈ 4.86 km/h
Relative Speed
If two people / objects are moving in same direction with speeds x km / h and y km / h (x >
y), then their relative speed would be (x - y) km / h
If two people / objects are moving in opposite direction with speeds x km / h and y km / h,
then their relative speed would be (x + y) km / h
Relative speed is the rate at which two moving bodies are separating from / coming closer to
each other. For example, if two persons are moving at 10 km/h and 20 km/h in opposite
directions, then their relative speed would be 10 + 20 = 30 km / h, i.e., the distance between
them after one hour would be 30 km. Similarly, if they were moving in the same direction,
their relative speed would be 20 - 10 = 10 km / h, i.e., the distance between them after one
hour would be 10 km.
Sample Problems
Question 1 : A runner can complete a 750 m race in two and a half minutes. Will he be able
to beat another runner who runs at 17.95 km/hr?
Solution: We are given that the first runner can complete a 750 m race in 2 minutes and 30
seconds or 150 seconds.
Also, we are given that the speed of the second runner is 17.95 km/hr.
Therefore, the first runner can beat the second runner.
Solution : We are given that two thirds of the 6 km was covered at 4 km / hr.
Question 3 : A postman traveled from his post office to a village in order to distribute mails.
He started on his bicycle from the post office at the speed of 25 km/hr. But, when he was
about to return, a thief stole his bicycle. As a result, he had to walk back to the post office on
foot at a speed of 4 km/hr. If the traveling part of his day lasted for 2 hours and 54 minutes,
find the distance between the post office and the village.
Solution : Let time taken by postman to travel from post office to village=t minutes.
According to the given situation, distance from post office to village, say d1=25/60*t km {25
km/hr = 25/60 km/minutes}
And distance from village to post office, say d2=4/60*(174-t) km {2 hours 54 minutes = 174
minutes}
Since distance between village and post office will always remain same i.e. d1 = d2
=> 25/60*t = 4/60*(174-t) => t = 24 minutes.
=> Distance between post office and village = speed*time =>25/60*24 = 10km
Question 4 : Walking at the speed of 5 km / hr from his home, a geek misses his train by 7
minutes. Had he walked 1 km / hr faster, he would have reached the station 5 minutes
before the actual departure time of the train. Find the distance between his home and the
station.
Solution : Let the distance between his home and the station be 'd' km.
Therefore, (d / 5) - (d / 6) = 0.2
=> d / 30 = 0.2
=> d = 6
Thus, the distance between his home and the station is 6 km.
Question 5 : Two stations B and M are 465 km distant. A train starts from B towards M at 10
AM with the speed 65 km/hr. Another train leaves from M towards B at 11 AM with the
speed 35 km/hr. Find the time when both the trains meet
Solution: The train leaving from B leaves an hour early than the train that leaves from M.
=> Distance covered by train leaving from B = 65 km / hr x 1 hr = 65 km
Now, the train from M also gets moving and both are moving towards each other.
Question 6 : A policeman sighted a robber from a distance of 300 m. The robber also noticed
the policeman and started running at 8 km/hr. The policeman also started running after him
at the speed of 10 km/hr. Find the distance that the robber would run before being caught.
Solution : Since both are running in the same direction, relative speed = 10 - 8 = 2 km / hr
Now, to catch the robber if he were stagnant, the policeman would have to run 300 m. But
since both are moving, the policeman needs to finish off this separation of 300 m.
=> 300 m (or 0.3 km)is to be covered at the relative speed of 2 km/hr.
Therefore, distance run by robber before being caught = Distance run in 0.15 hours
The time of running for both the policeman and the robber is the same.
Let the distance run by the robber be 'x' km at the speed of 8 km / hr.
=> 10 x = 8 (x + 0.3)
=> 10 x = 8 x + 2.4
=> 2 x = 2.4
=> x = 1.2
Question 7 : To cover a certain distance, a geek had two options, either to ride a horse or to
walk. If he walked one side and rode back the other side, it would have taken 4 hours. If he
had walked both ways, it would have taken 6 hours. How much time will he take if he rode
the horse both ways?
Solution : Time taken to walk one side + Time taken to ride one side = 4 hours
Time is taken to ride both sides = 2 x Time taken to walk one side = 6 hours
• If two trains are moving in same direction with speeds a km / hr and b km / hr, then
their relative speed would be |a - b| km / hr.
• If two trains are moving in different directions, i.e., coming towards each other or
going away from each other, with speeds a km/hr and b km/hr, then their relative
speed would be (a + b) km / hr.
• Time taken by a train, 't' meters long, to pass a stationary object of length 'l' meters
would be the time taken by the train to travel 't + l' meters. For example, to cover a
platform of 800 m, a train of length 200 m moving at the speed of 10 m / s would be
the time taken by the train to cover 800 + 200 = 1000 m at the speed of 10 m / s, i.e.,
1000 / 10 = 100 s.
• To pass a pole or a man or a post (or any stationary object with negligible length as
compared to the length of the train, like if the train is 500 m long and a pole is 1 m in
length), the time taken by the train would be the time it takes to travel the length of
the train. For example, if a train of length 100 m is moving at the speed of 10 m / s, it
would take 100 / 10 = 10 s to pass a pole / man / post.
• If two trains of lengths L1 and L2 are moving in the same direction with speeds S1
and S2, then the time required by faster train to overtake the slower train would be
the time taken to cover an equivalent distance of L1 + L2, with relative speed |S1 -
S2|, i.e., Time = (L1 + L2) / |S1 - S2|.
• If two trains of lengths L1 and L2 are moving in opposite directions with speeds S1
and S2, then the time required by the trains to cross each other completely would be
the time taken to cover an equivalent distance of L1 + L2, with relative speed (S1 +
S2), i.e., Time = (L1 + L2) / (S1 + S2).
• If two trains started moving towards each other at the same time with speeds S1 and
S2 respectively and after meeting, they take 'T1' and 'T2' seconds respectively, then
S1 : S2 = T2 1/2 : T11/2
o If the speed of the boat in still water is B km/hr and speed of the stream is S
km/hr,
1. Speed Upstream = B - S km / hr
2. Speed Downstream = B + S km / hr
Question 1 :
Therefore, time taken by the train to pass the man = 0.1 / 60 hour = (0.1 / 60) x 3600 sec = 6
sec
Question 2:
How long does a train 1000 m long moving at a speed of 90 km / hr would take to pass
through a 500 m long bridge?
Solution :
Here, time taken by the train to pass the bridge completely would be the time it takes to
cover 1000 + 500 = 1500 m at the speed of 90 km / hr = 90 x (5/18) = 25 m / sec.
Therefore, time required = 1500 / 25 = 60 sec = 1 minute
Question 3:
A man standing near a railway track observes that a train passes him in 80 seconds but to
pass by a 180 m long bridge, the same train takes 200 seconds. Find the speed of the train.
Solution :
=> The train covers L meters in 80 seconds and L + 180 meters in 200 seconds, with the same
speed.
=> L = 120
Question 4 :
Two trains 140 m and 160 m long are moving towards each other on parallel tracks with
speeds 40 km / hr and 50 km / hr respectively. How much time would they take to pass each
other completely ?
Solution :
Question 5 :
Two trains 140 m and 160 m long are moving in the same direction on parallel tracks with
speeds 40 km / hr and 50 km / hr respectively. How much time would the faster train require
to overtake the slower train ?
Solution :
Question 6 :
A 500 m long train takes 36 seconds to cross a man walking in the opposite direction at the
speed of 10 km / hr. Find the speed of the train.
Solution :
=> 50 = T + 10
=> T = 40 km / h
Therefore, speed of the train is 40 km / hr.
Question 7 :
A non - stop train started from Delhi towards Mumbai and at the same time, another non -
stop train started from Mumbai towards Delhi. If after meeting in Bhopal they took 9 and 16
hours respectively to reach their destinations, find the speed of the train that started from
Delhi, given that the speed of the train that started from Mumbai was moving at a speed of
90 km / hr.
Solution :
We know that for two trains starting at the same time, S1 : S2 = T2
1/2
: T1
1/2
Here, S2 = 90 km / hr
T1 = 9 hrs
T2 = 16 hrs
=> S1 : 90 = 4 : 3
=> S1 = 120 km / hr
Question 8 :
A boatman can row a boat upstream at 14 km / hr and downstream at 20 km / hr. Find the
speed of the boat in still water and speed of the stream.
Solution :
We are given that speed downstream, D = 20 km / hr and speed upstream, U = 14 km / hr
Another method :
Speed of the boat in still water = Speed of the stream + Speed Upstream = 3 + 14 = 17 km /
hr
Question 9 :
A boatman can row a boat at the speed of 5 km upstream and 15 km downstream. To cover
upstream he needs 2.5 hours and to cover downstream, he needs 1.5 hours. Find the speed
of the stream and speed of the boat in still water.
Solution :
We are given that the boatman covers 5 km upstream in 2.5 hours and 15 km downstream in
10 hours.
=> Speed upstream, U = 5 / 2.5 = 2 km / hr
Question 10 :
A man has to go from a port to an island and return. He can row a boat with the speed 7 km
/ hr in still water. The speed of the stream is 2 km / hr. If he takes 56 minutes to complete
the round trip, find the distance between the port and the island.
Solution :
Speed upstream = 7 - 2 = 5 km / hr
Speed downstream = 7 + 2 = 9 km / hr
Let the distance between the port and the island be D km. Also, we know that Time =
Distance / Speed
=> D = 3 km
Question 11 :
In a boat race, a person rows a boat 6 km upstream and return to the starting point in 4
hours. If the speed of the stream is 2 km / hr, find the speed of the boat in still water.
Solution :
Let the speed of the boat in still water be B km / hr.
=> 12 B = 4 (B - 2) (B + 2)
=> 3 B = B
2
-4
=> B
2
- 3 B - 4 =0
=> (B + 1) (B - 4) = 0
Question 12 :
A racer can row a boat 30 km upstream and 44 km downstream in 10 hours. Also, he can
row 40 km upstream and 55 km downstream in 13 hours. Find the speed of the boat in still
water and speed of the stream.
Solution :
D = 11 km / hr
U = 5 km / hr
Therefore, Speed of boat in still water = 0.5 x (D + U) km / hr = 0.5 x (11 + 5) = 8 km / hr
• Alligation is a rule that enables us to quickly calculate the price of a mixture, given
that it is a mix of two elements having different prices.
• Alligation Rule :
Here, mean price is the price of 1 unit of the mixture. Please note that in the above
formula, everything has to be calculated by considering one unit only. Another point
to be noted is that the mean price would always be between cheaper price and
dearer price.
• In questions of replacement where we initially have some quantity of pure element
(like petrol), and we keep on replacing a fixed portion of this pure liquid every time
with some other element (like water) thus making it a mixture, we apply the
following formula for finding the quantity of pure element after 'n' replacements : P x
[1 - (R / P)] n, where P is the initial quantity of pure element R is the quantity replaced
every time n is the number of replacements
Sample Problems
Question 1 :
From a vessel of 20 liters pure milk, 1 liter is taken out and replaced with water, so as to keep
the volume constant to 20 liters. This process is repeated 5 times. Find the percentage of
pure milk left in the vessel after 5 replacements.
Solution :
n = Number of replacements = 5
Therefore, percentage of pure milk left in the vessel after 5 replacements = (15.4756 / 20) x
100 = 77.378 %
Question 2 :
A dishonest shopkeeper mixed cheaper quality of rice, priced at Rs. 10 / KG with good
quality rice, priced at Rs. 25 / KG and sells the mixture at Rs. 15 / KG. Find the ratio in which
he mixes the two qualities of rice.
Solution :
Thus, ratio of quantities of cheaper and good quality rice = 10 : 5 = 2 : 1
• The money that the principal generates is called Interest. This is the money
generated as a result of borrowing/lending.
• Simple Interest is the interest calculated on the Principal amount, rather than being
calculated on cumulative amount.
Question 1 :
What would be the annual interest accrued on a deposit of Rs. 10,000 in a bank that pays 4
% per annum rate of simple interest ?
Solution :
Here, P = 10000, R = 4, T = 1
=> SI = P x R x T / 100
Question 2 :
A sum of money amounts to Rs. 28,000 in 2 years at 20 % simple interest per annum. Find
the sum.
Solution :
Here, A = 18000, T = 2, R = 20
=> A = P + SI
=> A = P + (P x R x T / 100)
=> A = P [1 + (R x T / 100)]
=> P = 20000
Question 3 :
A man borrowed a certain sum of money at the rate of 6 % per annum for the first two years
, 9% per annum for the next three years, and 14% per annum for the period beyond 5 years.
If he pays a total interest of Rs. 22,800 at the end of 9 years, find the amount he borrowed.
Solution :
=> SI for first 2 years + SI for next 3 years + SI for next 4 years = 22800
=> P = 24000
Question 4 :
At what annual rate of interest will a sum of money be thrice in 10 years?
Solution :
Amount = Principal + SI
If the sum of money would be thrice the principal after 10 years, the SI would be twice the
principal.
=> SI = 2 x P
=> (P x R x T / 100) = 2 X P
=> R x T / 100 = 2
=> R x T = 200
=> R x 10 = 200
=> R = 20 %
Question 5 :
The simple interest on a sum of money in 5 years at 12 % per annum is Rs. 400 less than the
simple interest accrued on the same sum in 7 years at 10 % per annum. Find the sum.
Solution :
Let the sum be P.
=> P = 4000
Question 6 :
A sum of Rs. 1000 was lent to two people, one at the rate of 5 % and other at the rate of 8
%. If the simple interest after one year is Rs. 62, find the sum lent at each rate.
Solution :
Let the sum lent at 5 % be P.
SI for 5 % + SI for 8 % = 62
=> 3 P = 1800
=> P = 600
Therefore, sum lent at 5 % = P = Rs. 600
Compound Interest
• The amount which is lent / deposited is called Principal
• The money that the principal generates is called Interest. This is the money
generated as a result of borrowing/lending.
• Compound Interest is the interest calculated on the cumulative amount, rather than
being calculated on the principal amount only.
• For finding the time period in which a sum of money will double itself at R % rate of
compound interest compounded annually, we generally use either of the following
two formulas :
1. Time, T = 72 / R Years
• When rate of interest is different for different years, say R1, R2, R3 and so on, the
amount is calculated as A = P [1 + (R1 / 100)] [1 + (R2 / 100)] [1 + (R3 / 100)] ...
Sample Problems
Question 1 :
Find the compound interest on Rs. 10,000 at 10% per annum for a time period of three and
a half years.
Solution:
Time period of 3 years and 6 months means for 3 years, the interest is compounded yearly
and for the remaining 6 months, the interest is compounded half yearly. This means that we
have 3 cycles of interest compounded yearly and 1 cycle of interest compounded half yearly.
[1 + ( {R/2} / 100 )]
=> Amount = 10000 [1 + 0.1]
3
[1 + 0.05]
(1.05)
Question 2 :
If Rs. 5000 amounts to Rs. 5832 in two years compounded annually, find the rate of interest
per annum.
Solution :
A = P [1 + (R / 100)]
=> 5832 = 5000 [1 + (R / 100)]
=> [1 + (R / 100)]
= 5832 / 5000
=> [1 + (R / 100)]
= 11664 / 10000
Question 3:
The difference between the SI and CI on a certain sum of money at a 10 % rate of annual
interest for 2 years is Rs. 549. Find the sum.
Solution :
Let the sum be P.
R = 10 %
n = 2 years
CI = A - P = P [1 + (R / 100)]
- P = 0.21 P
Question 4 :
A sum of Rs. 1000 is to be divided among two brothers such that if the interest being
compounded annually is 5 % per annum, then the money with the first brother after 4 years
is equal to the money with the second brother after 6 years.
Solution :
P [1 + (5 / 100)]
= (1000 - P) [1 + (5 / 100)]
=> P (1.05)
= (1000 - P) (1.05)
=> P = 524.38
Therefore, share of first brother = Rs. 524.38
Question 5 :
A sum of money amounts to Rs. 669 after 3 years and to Rs. 1003.50 after 6 years on
compound interest. Find the sum.
Solution :
= 1003.50
= 669
An investment doubles itself in 15 years if the interest is compounded annually. How many
years will it take to become 8 times?
Solution :
Area
Mensuration 2D is an important topic in Quantitative Aptitude, which mainly deals with
problems related to the perimeter and area of two-dimensional shapes such as triangles,
squares, rectangles, circles, parallelograms, etc. In quantitative Aptitude , this topic does not
have many variations and most of the questions are based on certain fixed formulas. The
ability to quickly and accurately calculate the area and perimeter of 2D shapes is an essential
skill required for various competitive exams, including aptitude tests.
Square
Area The region enclosed by the 2D figure is called the area.
units
Total Surface Square The sum of the area of all the surfaces of a three-
Area units dimensional object, including its base and top.
Square Unit A unit of measurement is used for area, where one unit is
Square
equivalent to the area of a square with sides of length 1
units
unit.
Triangle
• Perimeter = a + b + c
• Area
1. 2s = a + b + c
Area =
Rectangle
Square
Rhombus
Trapezium
Circle
• Perimeter = 2 π Radius
• Area = π (Radius) 2
• Length of an arc that subtends an angle θ at the center of the circle = (π x Radius x θ)
/ 180
• Area of a sector that subtends an angle θ at the center of the circle = (π x Radius2 x θ)
/ 360
Mensuration 2D Formulas
Circle
A = πr² (where r is the radius) C = 2πr (where r is the radius)
A = (b × h) / 2 (where b is the
Right-Angled P = a + b + c (where a, b, and c
length of the base, h is the
Triangle are the lengths of the sides)
length of the height)
Q1: Find the perimeter and area of an isosceles triangle whose equal sides are 5 cm and
height is 4 cm.
Solution:
Solution:
In questions like this, the diameter of the circle is lesser in length and breadth.
Here, the breadth Diameter of the circle = 7 cm
=> Radius of the circle = 3.5 cm
Therefore, area of the circle = π (Radius) 2 = π (3.5) 2 = 38.50 cm2
Q3: A pizza is to be divided into 8 identical pieces. What would be the angle subtended by
each piece at the center of the circle?
Solution:
By identical pieces, we mean that area of each piece is the same.
=> Area of each piece = (π x Radius2 x θ) / 360 = (1/8) x Area of circular pizza
=> (π x Radius2 x θ) / 360 = (1/8) x (π x Radius2)
=> θ / 360 = 1 / 8
=> θ = 360 / 8 = 45
Therefore, the angle subtended by each piece at the center of the circle = 45 degrees
Q4: Four cows are tied to each corner of a square field of side 7 cm. The cows are tied with
a rope such that each cow grazes the maximum possible field and all the cows graze in
equal areas. Find the area of the ungrazed field.
Solution:
For maximum and equal grazing, the length of each rope has to be 3.5 cm.
=> Area grazed by 1 cow = (π x Radius2 x θ) / 360
=> Area grazed by 1 cow = (π x 3.5 2 x 90) / 360 = (π x 3.5 2) / 4
=> Area grazed by 4 cows = 4 x [(π x 3.5 2) / 4] = π x 3.5 2
=> Area grazed by 4 cows = 38.5 cm 2
Now, area of square field = Side2 = 72 = 49 cm2
=> Area ungrazed = Area of field – Area grazed by 4 cows
=> Area ungrazed = 49 – 38.5 = 10.5 cm2
Q5: Find the area of the largest square that can be inscribed in a circle of radius ‘r’.
Solution:
The largest square that can be inscribed in the circle will have the diameter of the circle as
the diagonal of the square.
=> Diagonal of the square = 2 r
=> Side of the square = 2 r / 2 1/2
=> Side of the square = 2 1/2 r
Therefore, area of the square = Side2 = [2 1/2 r]2 = 2 r2
Q6: A contractor undertakes the job of fencing a rectangular field of length 100 m and
breadth 50 m. The cost of fencing is Rs. 2 per meter and the labor charges are Re. 1 per
meter, both paid directly to the contractor. Find the total cost of fencing if 10 % of the
amount paid to the contractor is paid as tax to the land authority.
Solution :
Total cost of fencing per meter = Rs. 2 + 1 = Rs. 3
Length of fencing required = Perimeter of the rectangular field = 2 (Length + Breadth)
=> Length of fencing required = 2 x (100 + 50) = 300 meter
=> Amount paid to the contractor = Rs. 3 x 300 = 900
=> Amount paid to the land authority = 10 % of Rs. 900 = Rs. 90
therefore, total cost of fencing = Rs. 900 + 90 = Rs. 990
Let the length, breadth, and height of the cuboid be ‘L’, ‘B’, and ‘H’ respectively.
• Volume = L x B x H
Cube
• Length of diagonal = a
Let the radius of the base and height of the right circular cylinder be ‘R’ and ‘H’ respectively.
• Volume = π R2 H
Let the inner radius of the base, outer radius of the base and height of the hollow right
circular cylinder be ‘r’, ‘R’ and ‘H’ respectively.
• Volume = π H (R2 – r2)
Cone
Let the radius of the base, slant height and height of the cone be ‘R’, ‘L’ and ‘H’ respectively.
• L2 = R2 + H2
• Volume = π R2 H / 3
Sphere
Let the radius of the sphere be ‘R’
• Volume = (4 / 3) π R3
• Surface area = 4 π R2
Hemisphere
• Volume = (2 / 3) π R3
• Curved Surface area = 2 π R2
Note: Please note that whenever it is mentioned to find “Surface Area”, we calculate the
total surface area.
Mensuration 3D Formulas
Here are some important formulas, we must consider while solving mensuration 3D
questions.
V = (1/3)πr²h
(where r is the CSA = πrl (where r is TSA = πr(r + l) (where r
Cone radius, h is the the radius, l is the is the radius, l is the
height) slant height) slant height)
Q1: Find the length of the largest rod that can be kept in a cuboidal room of dimensions 10
x 15 x 6 m.
Solution :
Largest rod would lie along the diagonal.
=> Length of largest rod = Length of diagonal of the room = (L2 + B2 + H2)1/2
=> Length of the largest rod = (102 + 152 + 62)1/2 = (100 + 225 + 36)1/2 = (361)1/2
=> Length of the largest rod = 19 m
Q2: Find the number of bricks of dimension 24 x 12 x 8 cm each that would be required to
make a wall 24 m long, 8 m high and 60 cm thick.
Solution :
Volume of 1 brick = 24 x 12 x 8 = 2304 cm 3
Volume of wall = 2400 x 800 x 60 = 115200000 cm 3
Therefore, number of bricks required = 115200000 / 2304 = 50000
Q3: A rectangular sheet of paper measuring 22 cm x 7 cm is rolled along the longer side to
make a cylinder. Find the volume of the cylinder formed.
Solution:
Q4: If each edge of a cube is increased by 10 %, what would be the percentage increase in
volume?
Solution:
Let the original edge length be ‘a’
=> Original volume = a3
Now, new edge length = 1.1 a
=> New volume = (1.1 a)3 = 1.331 a3
=> Increase in volume = 1.331 a3 – 1 a3 = 0.331 a3
Therefore, percentage increase int eh volume = (0.331 a3 / a3) x 100 = 33.1 %
Q5: Three metal cubes of edge lengths 3 cm, 4 cm, and 5 cm are melted to form a single
cube. Find the edge length of such a cube.
Solution:
Volume of new cube = Volume of metal generated on melting the cubes = Sum of volumes of
the three cubes
=> Volume of new cube = 3 3 + 4 3 + 5 3 = 216
=> Edge length of new cube = (216)1/3 = 6 cm
Q6: Find the length of a 1.25 m wide metal sheet required to make a conical machine of
radius 7 m and height 24 m.
Solution:
The sheet would be shaped into a cone.
=> Area of sheet = Area of conical machine
=> 1.25 x Length = π x R x L
=> 1.25 x Length = π x R x (72 + 242)1/2
=> 1.25 x Length = π x 7 x 25
=> Length = 440 m
Thus, 440 m long metal sheet is required to make the conical machine.
Q7: From a cylindrical vessel having a radius of the base of 7 cm and a height 6cm, water is
poured into small hemispherical bowls each of radius 3.5 cm. Find the minimum number
of bowls that would be required to empty the cylindrical vessel.
Solution :
Progressions
Progressions (or Sequences and Series) are numbers arranged in a particular order such that
they form a predictable order. By predictable order, we mean that given some numbers, we
can find the next numbers in the series.
• Sum of 'n' terms of an AP = 0.5 n (first term + last term) = 0.5 n [ 2a + (n-1) d ]
A sequence of numbers is called a geometric progression if the ratio of any two consecutive
terms is always the same. In simple terms, it means that the next number in the series is
calculated by multiplying a fixed number to the previous number in the series. This fixed
number is called the common ratio.
For example, 2,4,8,16 is a GP because ratio of any two consecutive terms in the series
(common difference) is same (4 / 2 = 8 / 4 = 16 / 8 = 2).
• If 'a' is the first term and 'r' is the common ratio,
A sequence of numbers is called a harmonic progression if the reciprocal of the terms are in
AP. In simple terms, a,b,c,d,e,f are in HP if 1/a, 1/b, 1/c, 1/d, 1/e, 1/f are in AP.
For two numbers, if A, G and H are respectively the arithmetic, geometric and harmonic
means, then
• A≥G≥H
• A H = G2, i.e., A, G, H are in GP
Sample Problems
Question 1 :
Find the nth term for the AP : 11, 17, 23, 29, ...
Solution :
Here, a = 11, d = 17 - 11 = 23 - 17 = 29 - 23 = 6
and so on ...
Question 2 :
Find the sum of the AP in the above question till first 10 terms.
Solution :
From the above question,
Question 3 :
For the elements 4 and 6, verify that A ≥ G ≥ H.
Solution :
A = Arithmetic Mean = (4 + 6) / 2 = 5
Therefore, A ≥ G ≥ H
Question 4 :
Find the sum of the series 32, 16, 8, 4, ... upto infinity.
Solution :
First term, a = 32
Question 5 :
The sum of three numbers in a GP is 26 and their product is 216. find the numbers.
Solution :
Let the numbers be a/r, a, ar.
=> (a / r) + a + a r = 26
=> a (1 + r + r2) / r = 26
Also, it is given that product = 216
=> a3 = 216
=> a = 6
=> 6 (1 + r + r2) / r = 26
=> (1 + r + r2) / r = 26 / 6 = 13 / 3
=> 3 + 3 r + 3 r2 = 13 r
=> 3 r2 - 10 r + 3 = 0
=> (r - 3) (r - (1 / 3) ) = 0
=> r = 3 or r = 1 / 3
Calenders
Calendars Formulas and Concepts:
What is Calendar?
A calendar is a system used to organize time into days, weeks, and months throughout the
year. It typically includes important dates, such as holidays or special events. There are
various types of calendars, depending on cultural or religious practices, but many follow the
same basic structure. This includes evaluating leap years, decoding the days of the week,
finding the day when another day is given or not given, and matching calendars for a
particular month. Understanding these concepts can help make planning and keeping track
of important dates much easier.
1. Odd Days:
To determine the day of the week for a specific date, we use the concept of “odd days”. Odd
days refer to the extra or remaining days in a given period that exceeded complete weeks.
For example, if a month has 30 days, there are two odd days because two days exceed four
complete weeks. It is important to understand this concept when working with calendars
and scheduling events on specific dates.
• Finding days from dates is based on calculating the number of odd days. By odd days,
we mean a number of days more than a complete number of weeks.
• For example, the number of days in a non-leap year = 365 365 mod 7 = 1 So, the
number of odd days in a non-leap year = 1
• Number of days in a leap year = 366 => Number of odd days in a leap year = 366 mod
7=2
• Number of odd days in 100 years (76 non-leap years + 24 leap years) = [(76 x 1) + (24
x 2)] mod 7 = (76 + 48) mod 7 = 124 mod 7 = 5 days
• Number of odd days in 200 years = (2 x Number of odd days in 100 years) mod 7 = 10
mod 7 = 3
January 3
February(ordinary/leap) (0/1)
March 3
April 2
May 3
June 2
July 3
August 3
September 2
October 3
November 2
December 3
2. Leap Year:
No. of days: 0 1 2 3 4 5 6
Solution:
To solve this problem, we need to count the number of days between January 1st, 2023, and
October 31st, 2023, and then find out what day of the week October 31st, 2023 falls on.
Total number of days between January 1st, 2023, and October 31st, 2023 = 31 + 28 + 31 + 30
+ 31 + 30 + 31 + 31 + 30 + 31 = 304 Now, we can find out what day of the week October 31st,
2023 falls on by adding 304 days to Sunday, which is the day of the week on January 1st,
2023. 304 divided by 7 leaves a remainder of 2, which means that 304 days after Sunday is
two days after Sunday, which is Tuesday. Therefore, October 31st, 2023 is on a Tuesday.
Q2. If March 1st, 2024 is a Saturday, what day of the week will be September 1st, 2024?
Solution:
To solve this problem, we need to count the number of days between March 1st, 2024, and
September 1st, 2024, and then find out what day of the week September 1st, 2024 falls on. A
total number of days = 31 + 30 + 31 + 30 + 31 + 31 + 1 = 185 Now, we can find out what day
of the week September 1st, 2024 falls on by adding 185 days to Saturday, which is the day of
the week on March 1st, 2024. 185 divided by 7 leaves a remainder of 3, which means that
185 days after Saturday is third days after Saturday, which is Tuesday. Therefore, September
1st, 2024 is on a Tuesday.
Type 2 Problems: Finding the day when another day is not given
Solution :
1600 will have 0 odd days. 300 years will have 1 odd day. Now, in the next 99 years, we
would be having 75 non-leap years and 24 leap years. => Number of odd days = (75 x 1) + (24
x 2) = 75 + 48 = 123 mod 7 = 4 odd days Total odd days till now = 1 + 4 = 5 Number of odd
days in January = 31 mod 7 = 3 Number of odd days in February (2000 is a leap year) = 29
mod 7 = 1 Number of odd days in March = 31 mod 7 = 3 Number of odd days till 14 April
2000 in the month of April= 14 mod 7 = 0 So, the total number of odd days = 5 + 3 + 1 + 3 =
12 mod 7 = 5 Thus, 14 April 2000 was Friday (odd days = 5 => Friday)
1600 will have 0 odd days. 300 years will have 1 odd day. Now, in the next 46 years, we
would be having 35 non-leap years and 11 leap years. => Number of odd days = (35 x 1) + (11
x 2) = 35 + 22 = 57 mod 7 = 1 odd days Total odd days till now = 1 + 1 = 2 N umber of odd days
in January = 31 mod 7 = 3 Number of odd days in February (1947 is a non – leap year) = 28
mod 7 = 0 Number of odd days in March = 31 mod 7 = 3 Number of odd days in April = 30
mod 7 = 2 Number of odd days in May = 31 mod 7 = 3 Number of odd days in June = 30 mod
7 = 2 Number of odd days in July = 31 mod 7 = 3 Number of odd days till 16 August 1947 = 16
mod 7 = 2 So, the total number of odd days = 2 + 3 + 0 + 3 + 2 + 3 + 2 + 3 + 2 = 20 mod 7 = 6
Thus, 16 August 1947 was Saturday (odd days = 6 => Saturday)
Clocks
Clocks Formulas and Concepts:
Here are some formulas, concepts, and short related to clocks that are commonly used in
quantitative aptitude exams:
Minute Spaces:
A typical analog clock has a circular face with twelve-hour markings, and 60-minute
markings placed around the circumference of the circle, called minute spaces.
• When it comes to telling time, clocks use two primary hands: the hour hand and the
minute hand.
• Hour hand, also known as the shorthand, is typically smaller and moves more slowly
than the minute hand.
• Meanwhile, the larger, faster-moving hand is called the minute hand or long hand.
• The markings on the face of a clock are 60 spaces, one each for a minute. Every hour,
the minute hand completes one round of 60 spaces and the hour hand completes
one full round every 12 hours.
Important Points and Shortcuts for Clock:
• In 60 minutes, the minute hand gains 55 spaces (also known as minute spaces) over
the hour hand. For example, if the initial time is 12:00, then after 1 hour, the minute
hand would cover 60 spaces whereas the hour hand would cover only 5 spaces. Thus,
the minute hand covers 55 spaces extra than the hour hand.
• The minute hand covers 360 degrees in 60 minutes. => In 1 minute, the minute hand
covers 360 / 60 = 6 degrees
• The hour hand covers 360 degrees in 12 hours. => In 1 hour, the hour hand covers 360
/ 12 = 30 degrees => In 1 minute, the hour hand covers 30 / 60 = 0.50 degrees
• The angle between the minute hand and the hour hand increases by 5.50 degrees
every minute. For example, after 2 minutes, angle made by the minute hand = 2 x 6 =
12 degrees and angle made by the hour hand = 2 x 0.50 = 1 degree => Angle between
the hour hand and the minute hand after 2 minutes = 12 – 1 = 11 degrees = 2 x 5.50
degrees
• In every hour, the minute hand and the hour hand coincide once.
• If the minute hand and the hour hand are in the same line, then the angle between
them is either 0 degree or 180 degrees.
• The angle between the minute hand and the hour hand is 180 degrees if they are 30
spaces apart, 90 degrees if they are 15 spaces apart, and 0 degrees if they are 0
spaces apart.
• If the clock shows time ahead of the actual time, it is said to be running fast. For
example, if the clock is showing 12:15 PM but it is actually 12:00 PM, then the clock is
said to be running 15 minutes fast.
• If the clock shows time behind the actual time, it is said to be running slow. For
example, if the clock is showing 2:15 PM but it is actually 2:30 PM, then the clock is
said to be running 15 minutes slow.
Below mentioned tables contain the angular values of the first ten minutes:
1 6°
2 12°
3 18°
4 24°
5 30°
6 36°
7 42°
8 48°
9 54°
10 60°
Solution:
At 3:00 PM, the angle made by the minute hand = 0 degree, and the angle made by the hour
hand = 3 x angle made by the hour hand in one hour = 3 x 30 = 90 degrees Now, in the next
20 minutes, angle made by the minute hand = 20 x angle made by the minute hand in 1
minute = 20 x 6 = 120 degrees and angle made by the hour hand = 20 x angle made by the
hour hand in 1 minute = 20 x 0.50 = 10 degrees => Angle made by the minute hand at 3:20
PM = 0 + 120 = 120 degrees => Angle made by the hour hand at 3:20 PM = 90 + 10 = 100
degrees Therefore, the angle between the hands of the clock at 3:20 PM = 120 – 100 = 20
degrees.
Another Method: At 3:00 PM, the angle made by the minute hand = 0 degree, and the angle
made by the hour hand = 3 x angle made by the hour hand in one hour = 3 x 30 = 90 degrees
=> Initial angle between the two hands = 90 degrees Now, we know that the difference
between the two hands of the clock increases every minute by 5.50 degrees. => Difference
between the hands of the clock after 20 minutes = 20 x 5.50 = 110 degrees Therefore, the
difference between the two hands at 3:20 PM = 110 – 90 = 20 degrees.
Q2: At what time between 3 PM and 4 PM would the two hands of the clock be together?
Solution:
At 3 PM, the hour hand would be at 15 spaces and the minute hand would be at 0 spaces.
The minute hand would have to cover these extra 15 spaces in order to meet the hour hand.
Now, 55 minutes are gained by the minute hand in 60 minutes. => 15 minutes would be
gained in (60 / 55) x 15 = 180 / 11 minutes Thus, the two hands of the clock meet at 180 / 11
minutes past 3 PM, i.e., around 3:16:22 PM.
Q3: How many times in a day the two hands of a clock coincide?
Solution:
Between 11 to 1, the hands of the clock coincide only once, i.e., at 12. At 12:00 AM and 12:00
PM, the hour hand and the minute hand do not coincide with each other So, every 12 hours,
they coincide 11 times. Therefore, the two hands of the clock coincide 22 times in a day.
Q4: At what time between 5 and 6 o’clock, do the minute and hour hands make an angle
of 34 degree with each other
Solution:
The angle between the minute hand and the hour hand at 5 o’clock is 150 degrees.
The angle between the hands becomes 34 degrees when the angle changes by 116 degrees
and 184 degrees, i.e. (150-34) and (150+34).
The angle changes by 5.5 degrees in 1 min.
The angle changes by 116 degrees in 1/5.5 x 116=21 1/11 min.
The angle changes by 184 degrees in 1/5.5 x 184=33 5/11 min.
Therefore the angle between the two hands is 34 degrees when the time is 5 hr 21 1/11 min,
and again at 5 hr 33 5/11 min.
For example, if we have two elements A and B, then there are two possible arrangements,
AB and BA.
Number of permutations when 'r' elements are arranged out of a total of 'n' elements
is n Pr = n! / (n - r)!.
For example, let n = 4 (A, B, C and D) and r = 2 (All permutations of size 2). The answer is
4!/(4-2)! = 12. The twelve permutations are AB, AC, AD, BA, BC, BD, CA, CB, CD, DA, DB, and
DC.
Combination It is the different selections of a given number of elements taken one by one,
or some, or all at a time. For example, if we have two elements A and B, then there is only
one way select two items, we select both of them.
Number of combinations when 'r' elements are selected out of a total of 'n' elements
is n C r = n! / [ (r !) x (n - r)! ]. For example, let n = 4 (A, B, C and D) and r = 2 (All combinations
of size 2). The answer is 4!/((4-2)!*2!) = 6. The six combinations are AB, AC, AD, BC, BD, CD.
n C r = n C (n - r)
NOTE: In the same example, we have different cases for permutation and combination. For
permutation, AB and BA are two different things but for selection, AB and BA are the same.
Sample Problems
Question 1: How many words can be formed by using 3 letters from the word "DELHI"?
Solution : The word "DELHI" has 5 different words. Therefore, required number of words
= 5 P 3 = 5! / (5 - 3)! => Required number of words = 5! / 2! = 120 / 2 = 60
Question 2: How many words can be formed by using the letters from the word "DRIVER"
such that all the vowels are always together?
Solution: In these types of questions, we assume all the vowels to be a single character, i.e.,
"IE" is a single character. So, now we have a total of 5 characters in the word, namely, D, R, V,
R, IE. But, R occurs 2 times. => Number of possible arrangements = 5! / 2! = 60 Now, the two
vowels can be arranged in 2! = 2 ways. => Total number of possible words such that the
vowels are always together= 60 x 2 = 120
Question 3 : In how many ways, can we select a team of 4 students from a given choice of 15
?
Solution : Number of possible ways of selection = 15 C 4 = 15 ! / [(4 !) x (11 !)] => Number of
possible ways of selection = (15 x 14 x 13 x 12) / (4 x 3 x 2 x 1) = 1365
Question 4 : In how many ways can a group of 5 members be formed by selecting 3 boys out
of 6 and 2 girls out of 5 ?
Question 5: How many words can be formed by using the letters from the word "DRIVER"
such that all the vowels are never together?
Solution: we assume all the vowels to be a single character, i.e., "IE" is a single character. So,
now we have a total of 5 characters in the word, namely, D, R, V, R, IE. But, R occurs 2 times.
=> Number of possible arrangements = 5! / 2! = 60 Now, the two vowels can be arranged in
2! = 2 ways. => Total number of possible words such that the vowels are always together =
60 x 2 = 120 Also, total number of possible words = 6! / 2! = 720 / 2 = 360 Therefore, total
number of possible words such that the vowels are never together = 360 - 120 = 240
Question 6: How many number greater than thousand can be formed from the digits 0, 1, 2,
3, 4 ?
Solution : In order to form a number greater than 1000 we should have only 5 digits. Since
we have 5 digits we cannot take 0 in starting position. For the first digit, we have 4 choices.
For the second digit, again we have 4 choices because we can include 0 from here onwards.
For the third digit, we have 3 choices. and for the fourth digit only left 2 choices. Total
numbers = 4x4x3x2= 96 Hence, only 96 numbers possible.
Question 7: In how many ways can 4 boys and 4 girls be seated around a circular table so
that no two boys are in adjacent positions?
Solution : If we first put 4 boys around the table, we can do this in 3! ways. Once the 4 boys
are placed, we have to place 4 girls around the same table. Now, we can see 4 vacant places
are there between all 4 boys so we can do in 4! ways. The total number of sitting
arrangements = 3! x 4!
Question 8: Out of the 11 points in a plane, 4 are collinear. How many straight line can be
formed ?
Solution : If all points were non-collinear then possible lines would have been 11C2. But, 4
points are collinear lie on the same line. So, they are all counted as a single line. Total
number of straight line = 11C2 - 4C2 + 1 = 11x10/2 - 4x3/2 + 1 = 55 - 6 + 1 = 50
Question 9: Twenty people attend a party and shake hands with one another. In how many
ways handshake is possible?
Solution : All people shake hands with one another except himself. The 1st person has 19
handshakes and 2nd also has 19 handshakes .........and so on 20 people x 19 handshakes.
and we know A handshake with B or B handshake with A, it is counted as 1 handshake. So,
total number of handshake = 20x19/2 = 190 Shortcut : for n people there are
always nC2 handshakes.
Question 10: How many different sums of money can be formed from the four type of notes
Rs 10, Rs 20, Rs 50 and Rs 100 ?
Solution : Type of notes = 4 So, total number of sum can be formed = 2 4 - 1 = 15
Question 11: Five chocolates of different flavors are to be distributed in three different
children such that any child get at least 1 chocolate. What is the maximum number of
different ways in which this can be distributed?
Solution : Acc. to question Chocolates can be distributed as [(3, 1, 1)(1, 3, 1)(1, 1, 3)] or [(2,
2, 1)(2, 1, 2)(1, 2, 2)] Total number of ways = 3 x 5C3 x 2C1 x 1C1 + 3 x 5C2 x 3C2 x 1C1 = 60 + 90 =
150
Question 12: Ram and his wife Sita both have five friends each. Ram has 2 boys and 3 girls.
Sita has 3 boys and 2 girls. In how many maximum number of different ways can they invite
2 boys and 2 girls such that two of them are Ram's friend and two are Sita's friend?
Solution : Selection can be done like that- i) 2 boys from Ram's friends and 2 girls from his
wife Sita's friends OR ii) 1 boy and 1 girl from Ram's friends and 1 boy and 1 girl from Sita's
friends OR iii) 2 boys from his wife Sita's friends and 2 girls from Ram's f riends Total number
of ways = 2C2.2C2 + 3C1 x 2C1 x 3C1 x 2C1 + 3C2.3C2 = 1 + 36 + 9 = 46 ways
Question 13: In how many ways 4 notebooks can be distributed to 5 students if each can get
any number of notebooks?
Solution : Since all the notebooks are identical or distinct we don't know. So, we take all are
distinct and it can be distributed in 5 4 ways.
Question 14: How many batting orders are possible for the Indian cricket team if there is a
squad of 16 to choose from such that Virat Kohli and Rohit Sharma are always chosen?
Solution: We need to select 9 players out of 14 players because two of them are already
selected. The selection of 11 players can be done in 14C9 ways. But batting order is also
required to calculate for these 11 players so arrangements can be done in 11! ways. Total
number of batting orders possible = 14C9 . 11!
Question 15: How many motor vehicle registration numbers plates can be formed for the
state Haryana having code like (HR 12Q 8702) with the digits 0, 1, 2, 3, 4, 5, 6 and contains
consonant at the alphabetical place(No digits being repeated).
Solution : For Haryana state number plate always contains HR in starting. Two consonants
already used in HR so remaining consonant = 21 -2 = 19. So, possible number of plates =
1(HR) x 7 x 6 x 19(consonants) x 5 x 4 x 3 x 2 = 95760
Probability
1. Experiment : An operation which can produce some well defined outcomes is called
experiment.
2. Random Experiment : An experiment in which all possible outcomes are known and
exact output can not be predicted in advance, is called a random Experiment.
3. Examples of performing a Random Experiment.
2. Tossing a fair dice. Dice is solid cube marked 1-6 numbers on it's faces.
Number on the upper face is considered as outcome of the event.
3. Drawing a card from a pack of well-shuffled cards. The pack of card consists -
▪ There are 4 face Cards in Each Suit namely Aces, Kings, Queens and
Jacks.
4. Sample Space : When we perform an experiment, then the Set S of all possible
outcomes is called Sample Space.
9.
13.
Sample Problems
Question 1 : Three unbiased coins are tossed. What is the probability that atmost one head
occurs ?
Solution : S = {HHH, HHT, HTH, THH, HTT, THT, TTH, TTT}
Favorable outcomes = {HTT, THT, TTH, TTT}
Total number of outcomes = 8
Number of favorable outcomes = 4
Required probability = 4 / 8 = 0.50
Question 2 : Find the probability of getting a red card when a card is drawn from a well
shuffled pack of cards.
Solution : Total number of outcomes = 52
Number of favorable outcomes = Number of red cards = 26
=> Required probability = 26 / 52 = 0.50
Question 3 : A bag contains 6 white and 4 black balls. Two balls are drawn at random from
the bag. Find the probability that both the balls are of the same color.
Solution : Outcome will be favorable if the two balls drawn are of the same color.
=> Number of favorable outcomes = 6 C 2 + 4 C 2 = 21
Total number of outcomes = 10 C 2 = 45
Therefore, required probability = 21 / 45 = 7 / 15
Question 4 : An unbiased die is tossed. Find the probability of getting an even number.
Solution : S = {1, 2, 3, 4, 5, 6}
Favorable outcomes = {2, 4, 6}
Required probability = 3 / 6 = 0.50
Question 5 : From a bag containing red and blue balls, 10 each, 2 balls are drawn at random.
Find the probability that one of them is red and the other is blue.
Solution : Total number of outcomes = 20 C 2 = 190
Number of favorable outcomes = 10 C 1 x 10 C 1 = 100
Therefore, required probability = 100 / 190 = 10 / 19
Question 6: If a card is picked up randomly from the pack of 52 cards. Find the probability
that it is a king or queen.
Solution : In a pack of 52 cards, 4 kings and 4 queens present.
Hence, the probability of getting a king or queen = 8/52
= 2/13
Question 7: In a throw of 2 dice, find the probability of getting one odd number and one
even number.
Solution : Total number of outcomes in throw of 2 dice = 36
Number of outcomes when one number is odd and one is even {(1, 2)(1, 4)(1, 6), (2, 1)(2,
3)(2, 5), (3, 2)(3, 4)(3, 6), (4, 1)(4, 3)(4, 5), (5, 2)(5, 4)(5, 6), (6, 1)(6, 3)(6, 5)}
Hence, Required probability = 18/36 = 1/2
Question 8: What will be the probability that a leap year chosen at random will have 53
Sundays.
Solution : A leap year has 366 days.
To find number of weeks = 366/7 = 52 weeks complete
2 days left in the year either of them can be Sunday.
(Saturday or Sunday), (Sunday or Monday)
So the possibility of 53 Sundays in a leap year = 2/7
Question 9: There are 5 women and 3 men applicants for a job.Only two out of eight are
selected for a job. The probability that at least one of the selected person will be a woman
is:
Solution : Selection can be done like that
First is a woman and second is a man
OR first is a man and second is a woman
OR both woman
Required probability = (5/8)(3/7) + (3/8)(5/7) + (5/8)(4/7)
= 15/56 + 15/56 + 20/56
= 50/56
= 25/28
Question 10: The probability that A can solve the problem is 3/4 and B can solve the
problem is 4/5. If both of them attempt the problem, then what is the probability that the
problem gets solved.
Solution : Probability that A can solve the problem = 1/4
and Probability that B can solve the problem = 1/5
Probability that problem not solved = 1/4 x 1/5 = 1/20
Hence, the problem is solved either by A or B = 1 - 1/20
= 19/20
Question 11: 200 students appeared for GATE and CAT examinations. 60% passed in GATE,
40% passed the CAT and 25% passed both. Find the probability that a student selected at
random has failed in both the examinations?
Solution : Number of students passed in GATE = 200 x 60% = 120
Number of students passed in CAT = 200 x 40% = 80
Number of students passed in both = 200 x 25% = 50
Number of students passed in either GATE or CAT = 120 + 80 - 50 = 150
Hence, Number of students failed in both = 200 - 150 = 50
Required probability = 50/200 = 1/4
Question 12: A box contains 40 bulbs out of which 4 are defective.Two bulbs are selected at
random from the box. What will be the probability that both bulbs found to be defective?
Solution : Both bulbs should come from the defective bulbs without replacement.
Required probability = 4/40 x 3/39 = 1/130
Question 13: Ten persons are seated round a circular table. What is the probability that
three friends always sit together?
Solution : Total number of ways = 9!
The total number of ways in which two people sit together = 7! x 3!
Required probability = 7! x 3!/ 9!
Question 14: A bag contains pens numbered from 1 to 17. A pen is drawn and replaced.
Then one more pen is drawn and replaced. What will be the probability that the first pen
drawn is even and the second one is odd.
Solution : In the first draw, we have 8 even numbered pens out of 15 and in second we have
9 odd numbered pens.
Required probability = 8/17 x 9/17
= 72/289
Question 15: If P(A)=2/3, P(B)=1/4, P(A ∩ B)=1/3 then find the P(A ∪ B)
Solution: P(A ∪ B)= P(A) + P(B) - P(A ∩ B)
=> 2/3 + 1/4 - 1/3
=> (8 + 3 - 4)/12
=> 7/12
Sure, here’s a table containing important formulas related to height and distance:
sin θ = Perpendicular / Hypotenuse
cos θ = Base / Hypotenuse
tan θ = Perpendicular / Base
cosec θ = Hypotenuse / Perpendicular
sec θ = Hypotenuse / Base
cot θ = Base / Perpendicular
sin^2 θ + cos^2 θ = 1
1 + tan^2 θ = sec^2 θ
1 + cot^2 θ = cosec^2 θ
Here, we can apply the formula Height = Distance / [cot(original angle) – cot(final angle)]
=> Height of the lighthouse = 100 / (cot 30 – cot 45) = 100 / (√3 – 1) = 50 √3 + 50 m
Q2: A 80 m long ladder is leaning on a wall. If the ladder makes an angle of 45 degrees
with the ground, find the distance of the ladder from the wall.
Solution :
Q3: There are two poles, one on each side of the road. The higher pole is 54 m high. From
the top of this pole, the angle of depression of the top and bottom of the shorter pole is
30 and 60 degrees respectively. Find the height of the shorter pole.
Solution :
Let AB and CD be the two poles.
Let AC = x m and CD = h m
Now, in triangle ABC,
tan 60 = AB / AC
=> √3 = 54 / AC
=> AC = 18 √3 m
Clearly, AC = DE = 18 √3 m
In triangle BED,
tan 30 = BE / DE
=> BE = DE tan 30
=> BE = 18 √3 / √3 m
=> BE = 18 m
=> CD = AE = AB – BE
=> CD = 54 – 18 = 36 m
Therefore, the height of the shorter pole = 36 m
Q4: From the top of a tower 100m high, a person observes that the angle of elevation of
the top of another tower is 60° and the angle of depression of the bottom of the tower is
30°. Then the height (in meters) of the second tower is.
Solution:
Q5: The angle of elevation of the top of a tower from point A on the ground is 45°. On
moving 20 meters toward the tower, the angle of elevation of the top of the tower
becomes 60°. Find the height of the tower (in meters).
Solution:
Let h be the height of the tower and x be the distance between the initial position and the
foot of the tower.
From the right-angled triangle AOB, where O is the foot of the tower, we have:
tan(45°) = h / x => h = x
From the right-angled triangle COB, where C is the new position of the observer, we have:
tan(60°) = h / (x – 20) => h = (x – 20) × √3
Equating both expressions for h, we get:
x = (x – 20) × √3
Simplifying, we get:
x = 20 × (√3 + 1)
Therefore, the height of the tower is:
h = x = 20 × (√3 + 1) meters, which is approximately 45.72 meters (rounded to two decimal
places).
Example –
Indices :
Example –
Let a number 2 3= 2×2×2= 8, then 2 is the base and 3 is indices.
An exponent of a number represents how many times a number is multiplied by itself.
Rules of surds :
When a surd is multiplied by a rational number then it is known as a mixed surd.
Example –
2√2, where 2 is a rational number and √2 is a surd. Here x, y used in the rules are decimal
numbers as follows.
1. n √x = x1/n √2 = 2 1/2
4. ( n √x) n = x (√2) 2 = 2
Rules of indices :
1. x0 = 1 20 = 1
2 x m × x n = x m +n 22 ×23= 25 = 32
3 x m ÷ x n = x m-n 23 ÷ 22 = 23-2 = 2
4 (x m) n = x m ×n (23) 2 = 23×2 = 64
5 (x × y) n = x n × y n (2 × 3) 2= 22 × 32 =36
6 (x ÷ y) n = x n ÷ y n (4 ÷ 2) 2= 42 ÷ 22 = 4
Other Rules :
Some other rules are used in solving surds and indices problems as follows.
Solution –
An answer is an option (d)
Explanation -
3√25= (25) 1/3 = 2.92401773821... which is irrational So it is surd.
Question-2 :
Find √√√3
Solution –
An answer is an option (d)
Explanation -
((3 1/2) 1/2) 1/2) = 31/2 × 1/2 ×1/2 = 3 1/8 according to rule number 5 in indices.
Question-3 :
If (4/5) 3 (4/5) -6= (4/5) 2x-1, the value of x is
a) -2 b)2 c) -1 d)1
Solution –
The answer is option (c)
Explanation -
LHS = (4/5) 3 (4/5) -6= (4/5) 3-6 = (4/5) -3
RHS = (4/5) 2x-1
According to question LHS = RHS
⇒ (4/5) -3 = (4/5) 2x-1
⇒ 2x-1 = -3
⇒ 2x = -2
⇒ x = -1
Question-4 :
34x+1 = 1/27, then x is
Solution –
34x+1 = (1/3) 3
⇒34x+1 = 3-3
⇒4x+1 = -3
⇒4x= -4
⇒x = -1
Question-5 :
Find the smallest among 2 1/12, 3 1/72, 41/24,61/36.
Solution –
The answer is 3 1/72
Explanation –
As the exponents of all numbers are infractions, therefore multiply each exponent by LCM of
all the exponents. The LCM of all numbers is 72.
2(1/12 × 72) = 26 = 64
3( 1/72 ×72) = 3
4(1/24 ×72) = 43 = 64
6 (1/36 ×72) = 62 = 36
Question-6 :
The greatest among 2 400, 3300,5200,6200.
Explanation –
As the power of each number is large, and it is very difficult to compare them, therefore we
will divide each exponent by a common factor(i.e. take HCF of each exponent).
logical-reasoning
Inequality Reasoning
It is an inequality which is true for some variables or for a particular condition but not true
for all values of variables. And the solution of inequality consists of only real numbers as the
term ” Less than or Greater than” are not defined for establish a certain relation. For
example;
Q1. Which of the following statements prove that ‘W > F’ is definitely true?
Solution: III
Q2. Which of the following statements prove that ‘K > S’ is definitely true?
f) Coded Inequality:
In case of coded inequality, questions consists of a couple of statements with some logical
and arithmetic relationship between them. Such type of Inequality followed by a couple of
conclusions and you’ll have to find out which conclusion follows the given statements. For
example;
Q. In these questions, the relationship between different elements is shown in the
statement. The statement is followed by two conclusions. Choose the correct answer on
the basis of the information given below.
In the following questions, the symbols %, @, #, &, $ are used. All the symbols define the
following meanings.
Q. Statements:
O $ M; P # M; R % P;
Conclusions:
I) P % O
II) P # O
Ans: c
Solution:
Step 1 – Decode the given symbols as shown below:
Symbols % @ # & $
Step 2 – Now decode the given statements with the help of the above table:
O ≥ M; P = M; R < P.
After arranging, we have;
R <P =M ≤O
Step 3 – Now based on the given statement; we can conclude that either O > P or O = P will
be true.
So, the correct answer is c.
Alphanumeric Series
Alphanumeric series is formed by combining both alphabets and numbers. Alphanumeric
series sometime include symbols such as @,&,#, etc. So we can say that the Alphanumeric
series is the combination of alphabets, numbers and symbols. This is an important topic that
needs to be covered for the Logical Reasoning section of the Aptitude test asked in the
competitive examinations.
Here are some Alphanumeric series Questions and Answers which will make your concept
more strong. So let us start practicing by going through the following questions.
To solve Alphanumeric Questions, follow the tips and tricks given below:
1. For missing series: When a series with a pattern is given containing alphabets and
numbers. The candidate is asked to find the missing number in the blank space.
3. Alphabetic Series: This type of Series only contains English Alphabets set in a
particular pattern. No numbers or symbols are used here.
4. Numerical Series: This type of Series only contains numbers, arranged in a specific
pattern. Questions based on this type is usually asked in the form of rearrangement
of numbers in ascending, descending or any other order.
Sample Questions on Alphanumeric Series
Direction (1-5): Answer the following questions based on the arrangement given below:
Q1. If the symbols followed by consonants interchange their positions within the group,
then which element is third from the right end?
B@C7NR%5$G6KM&4S #PU5
(a) U
(b) #
(c) P
(d) 5
(e) None of these
Answer: b
Explanation:-
Given series – B @ C 7 N R % 5 $ G 6 K M & 4 S # P U 5
After operation – B C @ 7 N R % 5 G $ 6 K M & 4 S P # U 5
So, third element from the right end = #
Q2. Based on the given arrangement which of the following groups should be next?
BC7 R5$ 6M&?
(a) N5G
(b) KS#
(c) SPU
(d) C4H
(e) 4SP
Answer: c
Explanation:-
Given series – B @ C 7 N R % 5 $ G 6 K M & 4 S # P U 5
So, SPU is the correct answer.
Q3. Which of the following is second to the left of the twelfth from the right end if all the
symbols are dropped?
(a) F
(b) C
(c) 7
(d) M
(e) None of these
Answer: b
Explanation:-
Given series – B @ C 7 N R % 5 $ G 6 K M & 4 S # P U 5
After dropping symbols – B C 7 N R 5 G 6 K M 4 S P U 5
So, 12th from the right end – N
Hence, second to the left of N is C.
Q4. If the numbers which are preceded by letters interchanged their positions and those
letters were changed to the next letter in the alphabetical series, then which of the
following elements are the fourth from the left end and the eleventh from the right end?
(a) F and 7
(b) 6 and D
(c) A and $
(d) D and 6
(e) None of these
Answer: d
Explanation:-
Given series – B @ C 7 N R % 5 $ G 6 K M & 4 S # P U 5
After operation – B @ 7 C N R % 5 $ 6 G K M & 4 S # P 5 U
After changing – B @ 7 D N R % 5 $ 6 H K M & 4 S # P 5 V
So, fourth from the left end and eleventh from the right end – D and 6
Therefore, option d is the correct answer.
Q5. How many letters are there which are preceded by a symbol and followed by a
number within the group in the given arrangement?
(a) None
(b) One
(c) Two
(d) Three
(e) More than three
Answer: c
Explanation:-
Given series – B @ C 7 N R % 5 $ G 6 K M & 4 S # P U 5
So, there are two letters (@C7, $G6). Therefore option c is the correct answer.
Direction (6-10): Study the following digit-letter-symbol sequence carefully and answer the
questions given below.
Q6. If all the digits are dropped, which of the following is third to the right of the eleventh
from the right end?
*E7O&6^4Z5*39T!D#2$U
(a) ^
(b) 8
(c) Z
(d) &
(e) T
Answer: c
Explanation:-
Given Series – * E 7 O & 6 ^ 4 Z 5 * 3 9 T ! D # 2 $ U
After dropping digits – * E O & ^ Z * T ! D # $ U
So, eleventh from the right end – O
Therefore, third to the right of O – Z
Q7. What is the 7th element to the left of the element that is 9th from the right end?
(a) &
(b) C
(c) 2
(d) #
(e) $
Answer: a
Explanation:-
Given Series – * E 7 O & 6 ^ 4 Z 5 * 3 9 T ! D # 2 $ U
9th from the right end – 3
So, the 7th element to the left of 3 – &
Hence, option a) is the correct answer.
Q8. How many digits are there which are preceded by a symbol and followed by a letter?
(a) 0
(b) 1
(c) 2
(d) 3
(e) More than 3
Answer: b
Explanation:-
Given Series – * E 7 O & 6 ^ 4 Z 5 * 3 9 T ! D # 2 $ U
So, there is one digit (4). Hence, option (b) is the correct answer.
Q9. If all the symbols are removed then what will be the element which is 9th from the
left?
(a) 4
(b) 7
(c) 1
(d) 9
(e) None of the above
Answer: d
Explanation:-
Given Series – * E 7 O & 6 ^ 4 Z 5 * 3 9 T ! D # 2 $ U
After dropping symbols – E 7 O 6 4 Z 5 3 9 T D 2 U
So, 9th from the left end – 9
Hence, option (d) is the correct answer.
Q10. If all the digits are arranged in decreasing order from the left, how many digits will
have their positions unchanged?
(a) 0
(b) 1
(c) 2
(d) 3
(e) More than 3
Answer: c
Explanation:-
Given Series – * E 7 O & 6 ^ 4 Z 5 * 3 9 T ! D # 2 $ U
After operation – * E 9 O & 7 ^ 6 Z 5 * 4 3 T ! D # 2 $ U
So, 2 digits that is 5 and 2 has not changed. Hence, option (c) is the correct answer.
(a) JDI
(b) 4I6
(c) 6IJ
(d) 6#J
(e) None of these
Answer: d
Explanation:-
Given series – Y Z # 8 F H 7 $ 3 M O & 9 7 G Q % K X @ 4 6 J I # D T S 8 $
Therefore, option (d) is the correct answer.
Q12. How many such numbers are there in the given series which are immediately
preceded or followed by a consonant?
(a) Three
(b) Four
(c) More than five
(d) Five
(e) None of these
Answer: c
Explanation:-
Given series – Y Z # 8 F H 7 $ 3 M O & 9 7 G Q % K X @ 4 6 J I # D T S 8 $
So, there are six such numbers which are immediately preceded or followed by a consonant.
Hence, option (c) is the correct answer.
Q13. If all the symbols are dropped from the series then which element will be fifth to the
right of the one which is fourteenth from the right end of the new arrangement?
(a) G
(b) K
(c) 4
(d) X
(e) None of these
Answer: d
Explanation:-
Given series – Y Z # 8 F H 7 Misplaced &Misplaced &
After dropping symbols – Y Z 8 F H 7 3 M O 9 7 G Q K X 4 6 J I D T S 8
So, fourteenth from the right end – 9
Then, fifth to the right of 9 – X
Hence, option (d) is the correct answer.
Q14. Which of the following element is 17th to the right of the one which is 24th from the
right end of the given arrangement?
(a) I
(b) J
(c) #
(d) T
(e) None of these
Answer: a
Explanation:-
Given series – Y Z # 8 F H 7 Misplaced &Misplaced &
So, 24th from the right end – 7
Then, 17th to the right of 7 – I
Hence, option (a) is the correct answer.
Q15. How many such symbols are there in the given series which are immediately
preceded by a consonant but not immediately followed by a consonant?
(a) Four
(b) One
(c) Three
(d) Two
(e) None of these
Answer: a
Explanation:-
Given series – Y Z # 8 F H 7 $ 3 M O & 9 7 G Q % K X @ 4 6 J I # D T S 8 $
So, there are four symbols. Hence, option (a) is the correct answer.
Direction (16-20): Study the following digit-letter-symbol sequence carefully and answer
the questions given below.
Q16. How many such symbols are there in the above arrangement each of which is either
immediately followed by a number or immediately preceded by a letter, but not both?
(a) None
(b) One
(c) Two
(d) Three
(e) More than 3
Answer: d
Explanation:-
Given Series – M Z 3 + T # & I H N % 8 $ J Q 1 L M * @ X R 6 4 # D F +
So, there are three symbols in the above arrangement. Hence, option (d) is the correct
answer.
Q17. Four of the following five are alike in a certain way. Which one does not belong to the
group?
(a) 3 Z D F
(b) # T 6 4
(c) M T + 4
(d) # I 6 X
(e) + # 3 D
Answer: e
Explanation:-
Given Series – M Z 3 + T # & I H N % 8 $ J Q 1 L M * @ X R 6 4 # D F +
In all other groups, the 3rd and 4th elements occupy the same positions from the right in the
given arrangement as the first and 2nd elements respectively from the left end.
Hence, option (e) is the correct answer.
Q18. 3 Z T is to & H % in the same way as Q J L is to?
(a) X R *
(b) * XM
(c) @ * R
(d) @ X L
(e) None of these
Answer: b
Explanation:-
Given Series – M Z 3 + T # & I H N % 8 $ J Q 1 L M * @ X R 6 4 # D F +
In each pair, the 1st, 2nd and 3rd elements of the 1st term move 4, 7 and 1 step forward
respectively to give the corresponding elements of the 2nd term.
Hence, option (b) is the correct answer.
Q19. How many such numbers are there each of which is either immediately followed by a
symbol or immediately preceded by a letter, but not both?
(a) None
(b) One
(c) Two
(d) Three
(e) More than three
Answer: e
Explanation:-
Given Series – M Z 3 + T # & I H N % 8 $ J Q 1 L M * @ X R 6 4 # D F +
Hence, there are four numbers. So, option (e) is the correct answer.
Q20. How many such letters are there, each of which is either immediately followed by a
number or immediately preceded by a symbol but not both?
(a) None
(b) One
(c) Two
(d) Three
(e) More than 3
Answer: e
Explanation:-
Given Series – M Z 3 + T # & I H N % 8 $ J Q 1 L M * @ X R 6 4 # D F +
Hence, there are four letters which is either immediately followed by a number or
immediately preceded by a symbol but not both. Therefore option (e) is the correct answer.
Directions (1-3): In each of the following letter series, some of the letters are missing
which are given in that order as one of the alternatives below it. Choose the correct
alternative.
Q1. g b _ b b c _ c _ g b _ g b _ b
(a) ccggc
(b) cbgbc
(c) cgcgc
(d) bccgb
Sol. Ans.(c)
The pattern of the series is as follows
gbc / b / bcg /c / cgb / g / gbc / b.
Q2. _ c _ b _ c b c c _ b c b _ c b b
(a) cccbb
(b) cbcbb
(c) bcbcb
(d) bcbbc
Sol. Ans.(d)
The series is bcc/bbc/bcc/bbc/bcc/bb.
Q3. c _ d e e _ e c e _ c e c e d _ e c e c _ d c e
(a) ecdee
(b) ddcee
(c) dcced
(d) cdeec
Sol. Ans.(a)
The series is ced/ee/ceced/ceced/ee/ceced/ce.
Thus, the pattern ‘ceced/ceced/ee’ is repeated.
Q4. Find the term which does not fit into the series:
1FG, 5JF, 9NE, 15RD, 17VC
(a)15RD
(b) 17VC
(c) 9NE
(d) 5JF
Directions (5-15): In each of the following questions, a letter-number series is given with
one or more terms missing as shown by (?). Choose the missing term out of the given
alternatives.
Directions
There are many different types of questions that are asked from the Direction and
Distance topic. Those questions are generally related to the direction and distance
between two or more persons, products or elements, etc. Thus, many different types of
questions can be formed based on the position of any person, thing, or product. In this
topic, we will discuss how the concepts of Direction and Distance work. Let us understand, it
with the help of the important rules:
There are four cardinal directions North (N), South (S), East (E), and West (W), and the four
ordinal directions (also called the intercardinal direction) North-East (NE), North-West
(NW), South-East (SE), and South -West (SW).
• In case a person is facing east, on taking the left he will face towards the north and
on taking the right turn he will face towards the south.
• In case a person is facing west, on taking the left he will face towards the south and
on taking the right turn he will face towards the north.
• In case a person is facing north, on taking the left he will face towards the west and
on taking the right turn he will face towards the east.
• In case a person is facing south, on taking the left he will face towards the east and
on taking the right turn he will face towards the west.
Q1. Ankita is in the east of Devesh and in the north of Esha. If Gagan is in the south of Esha,
then Ankita is in which direction with respect to Gagan?
Solution:
let us understand it with the help of diagrams.
Let us start with Esha, and then from there, we can draw others’ positions as well.
Hence, Ankita is towards the North of Gagan.
b) Degree-based questions:
• If a person goes to his left side, he will go towards the anti-clockwise direction.
• If a person goes to his right side, he will go towards the clockwise direction.
* In the case of moving towards the left or right side, we assume that the movement is at an
angle of 90 degrees.
1) East
2) West
3) North
4) South-East
5) North-East
Answer: 4
Solution:
According to the question, she turns 135 degrees in the anticlockwise direction and then
another 180 degrees in the same direction
Therefore, she turns total (135 + 180) = 315 degree anticlockwise
In the primary stage if she faces north then, finally she will face the South-East direction.
3Q. Direction: Study the following information carefully and answer the questions given
below.
In a coded language:
• A@B (11) means A is 11m in the north of B
1) North
2) West
3) North-west
4) East
5) South-east
Answer: 2
Solution:
In the morning, as the sun rises from the east, so the shadow of any objects formed is in the
west direction.
At noon, or 12 pm, when the sun is exactly on top , no shadow will form.
In the evening , as the sun sets in the West , so the shadow of any objects formed is in the
East.
Keeping these directions in mind, there are some types of questions which come into the
exams, which we will discuss one by one.
Solution:
Now, according to the question, and the directions in mind,
So, from the above image it is clear that ram is 7 km away from the starting point.
Q2: Sam started walking from point A towards East and walked for 6 km, then he turned to
the left and walked for 8 km to reach point B. How far was he from the starting point?
Solution:
Now, according to the question,
= 10 km
Solution:
The sun rises in the East (E) in the morning.
As the shadow of Mahesh falls to his right.
So he must be facing South.
Hence, the answer is the south.
Q4: M is in the East of N, which is in the North of S. If P is in the South of S, then in which
direction of M, is P?
Solution:
Let us understand through the diagram , which will clear the question more.
So, P is south-west of M.
Q5: You are facing south. You turn right and walk 20 m , then you turn right and walk 10m.
Then turn left and walk for 10 m. then turn right and walk for 20 m. Then you turn right
again and walk 60 m. In which direction do you in from the starting point?
Solution:
First we have to make a sketch of all the given data and then we will analyze it to get the
answer or to know the direction,
we start facing south, and after that follow the paths given in question above,
Now, we have to find a direction from the starting point.
Here , we will also find the distance from start to F , distance = √(302 + 302 ) = 30√2.
and the direction is N-E ( north east).
Data Sufficiency
Data Sufficiency in Logical Reasoning is an important topic to prepare for competitive exams
in India. It tests the logical, analytical and thinking capacity or ability of a candidate.
Candidates need to practice the questions for scoring well in the examination
The following article contains various kinds of Data Sufficiency Questions. Candidates must
go through the article to practice all the important questions of the topic.
Tips and Tricks to Solve Data Sufficiency Questions
1. Read the entire question carefully before attempting it. Then go through the options to
find the correct answer.
2. Use the information given in the question and avoid making any assumptions.
3. Both the statements must be carefully read and then a make a conclusion.
Given below is a question and a set of statements. We have to find which of the given
statements is required to reach the conclusion.
Solution : If we consider statement 1 only, we can safely conclude that B is the tallest of all. If
we consider statement 2 only, we can safely conclude that A is the shortest of all. Therefore,
statement 1 alone is sufficient to answer the question.
Answer: IV) Both statement 1 and statement 2 even together are not sufficient
Rule No 1:
To find the total number of persons/things of a single row, when ranks of one person/things
is/are given from both sides of the row.
Formula: Total number of persons/things = Rank from Left end + Rank from Right end – 1
Example:
In a row of girls, the position of Radhika is 38th from the left side of the row, and the
position from the right side of the row is 19th. Find the total number of girls in that row.
Solution:
Total no. of person= (The position of Pooja from the left end + the position of Pooja from
the right end) – 1
Rule No. 2:
To find the rank or position of a person/thing from the opposite side, when rank or position
from one side and total number of persons/things are given.
Formula: Position of a person from the opposite ends= (Total number of persons of that
row – Position of the same person/thing from the given side) + 1
Example:
In a row of 31 persons, the position of Vicky from the left side of the row is 8th. What is the
position of Vicky from the right side of that row?
Solution:
The position of Vicky from the right side is = (The total number of persons of that row –
Position of Vicky from the left side) +1 = (31-8) + 1 = 24
Rule No. 3:
To find the number of persons/things after or before a person/thing whose rank from
another side is given –
Number of persons/things after or before the given person/thing in a row = Total no. of
persons/things – Position of the same person/thing from another side
Example:
In a billing line of 32 persons, the position of Hafiz from the front side of the row is 14th.
Find the number of persons after Hafiz in that row?
1. 17
2. 16
3. 14
4. 18
5. 13
Ans: 4
Solution:
Number of persons after Hafiz = Total number of persons – Position of Hafiz from the front
side
⇒ Number of persons after Hafiz in the row = 32 – 14 = 18
Rule No. 4:
To find the total number of persons/things in a row, when ranks of two persons/things and
number of the persons/things who are sitting between those two persons/things are given.
A. Case of Overlapping:
When the number of persons/things –> (position of a person/thing from the right side +
position of another person/thing from the left side) is greater than the total number of
persons/things
In a row of 22 boys. Ronak is 19th from the left side of the row and Shanky is 9th from the
right side of the row. Find out the number of persons sitting between them?
Solution:
When Total number of persons/things is greater than the (position of one person from the
right side /thing + position of another person/thing from the left side).
Number of persons between two persons/articles = Total number of persons/articles –
(Sum of positions of two different persons/articles from both sides)
Example:
In a row of 85 persons. Manya is 24th from the left side of the row and Rashi is 19th from
the right side of the row. Find out the number of persons sitting between them?
Solution:
Rule No. 5:
When in a single row, the positions of two persons are given and their positions are
interchanged and after interchanging the position of first person is given from the same side
as before interchanging.
Formula:
Total Number of persons = Sum of the particular positions of a place (different person)
from both sides – 1
Example:
In a row of persons, Ashish is 11th from the left and Salman is 20th from the right. If they
interchange their positions, then Ashish becomes 17th from the left. Find out the total
number of persons of that row.
Solution:
Total Number of persons = Sum of the particular positions of a place (different person)
from both sides – 1
Positions of Salman before change + position of Ashish after change – 1
20 + 17 – 1 = 36
b) To find the new position of the second person from the same side as before
interchanging:
Formula:
The position of the second person from the same side as before interchanging = The position
of the second person from the same side before interchanging + (position of the first person
after interchanging – position of the first person before interchanging from the same side
Example:
In a row of persons, Ashish is 11th from the left and Salman is 20th from the right. If they
interchange their positions, then Ashish becomes 17th from the left. Then, what will be the
new position of Salman from the right end?
Total no of persons in a row = Position from the right end + Position from the left end – 1
= 20 + 17 – 1 = 35
Salman’s position from right end = Total persons – Ashish’s position from the left end + 1
= 36 – 10 + 1 = 27
Hence, in the new arrangement, the position of Salman is 27th from the right end.
Formula:
Number of persons between two persons = Difference in the positions of a person (same
person) whose position from same side before and after interchanging is given – 1
Example:
In a row, Avni is 14th from the left end and Tanmay is 13th from the right end of the row. If
they interchanged their positions Avni becomes 19th from the left end. Find the number of
persons between them.
Solution:
Number of persons between Avni & Tanmay = (Position of Avni from left after interchanging–
Position of Avni from left before interchanging) – 1
⇒ Number of persons between Avni & Tanmay is = (19 – 14) – 1 = 5 – 1 = 4
Rule No. 6:
In a row, if the positions of two different persons are given from opposite sides of the row
and a third person is sitting exactly in the middle of that two person. If total number of
persons is to be calculated, then there are will be two conditions –
a) When the position of the third person is to the left side of that person whose position was
given from the right side. Or when the position of the third person is to the right side of that
person whose position was given from the left side.
b) When the position of the third person is to the left side of that person whose position was
given from the left side. Or when the position of the third person is to the right side of that
person whose position was given from the right side.
Example:
In a row there are some persons, the position of Nishant from the left side of the row is 16th
and position of Shikha from the right side of the row is 11th. If Mahesh is sitting just in the
middle of Nishant and Shikha and there are two persons between Mahesh and Nishant. Find
the total number of persons in the row?
Solution:
A) If Mahesh is to the left of Shikha, then the number of persons in the row will be:
Shortcut Rules:
⇒ Position of 1st person+ Position of 2nd person + twice of the number of people between
the third person and any of them (which was given) + 1
⇒ Position of Nishant + Position of Shikha + twice of the number of people between the
third person and any of them (which was given) + 1
⇒ 16 + 11 + (2 × 2) + 1 = 32
B) If Mahesh is to the right of Shikha, then the number of persons in the row will be:
Shortcut Rules:
⇒ Position of 1st person+ Position of 2nd person + twice of the number of people between
the third person and any of them (which was given) – 3
⇒ Position of Nishant + Position of Shikha – twice of the number of people between the
third person and any of them (which was given) – 3
⇒ 16 + 11 – (2 × 2) – 3 = 20
• Position of a person from the opposite ends = (Total number of persons of that row –
Position of the same person/thing from the given side) + 1
• Number of persons/things after or before the given person/thing in a row = Total no.
of persons/things – Position of the same person/thing from another side
• When the number of persons/things –> (position of a person/thing from the right
side + position of another person/thing from the left side) is greater than the total
number of persons/things (This is a case of overlapping). Then the formula will be:
Number of persons between them = (Sum of positions of two different
persons/things from both sides) – (Total number of persons/things) – 2
• When total number of persons/things is greater than the (position of one person
from the right side /thing + position of another person/thing from the left side) (This
is a case of not-overlapping). Then the formula will be: Number of persons between
two persons/articles = Total number of persons/articles – (Sum of positions of two
different persons/articles from both sides)
• In a row, if the positions of two different persons are given from opposite sides of the
row and a third person is sitting exactly in the middle of that two person. If total
number of persons is to be calculated, then there are will be two conditions –
• When the position of the third person is to the left side of that person whose
position was given from the right side. Or when the position of the third person is to
the right side of that person whose position was given from the left side.
• When the position of the third person is to the left side of that person whose
position was given from the left side. Or when the position of the third person is to
the right side of that person whose position was given from the right side.
• Q1. In a row of kids all are facing North, K is 35th from the left end while R is
20th from the right end and 9th to the right of E. If K sits second to the left of
E then, find out the total number of persons of the row?
• 1) 65
• 2) 67
• 3) 68
• 4) Can’t be determined
• 5) None of these
• Ans: 1
• Solution:
• Here, R is 20th from the right end and 9th to the right of E. Therefore, E is (20+9)
= 29th from the right ends.
• Again, given that, K is 35th from the left ends and K sits second to the left of E.
Therefore, E is (35+2) = 37th from the left ends.
• = E’s position from the right ends + E’s position from the left ends – 1
• = 29 + 37 – 1 = 65
• Q2. Among Ayush, Jindal, Ketan, Urvashi and Mahi, each one of them having
different height, Urvashi is taller than Ketan. Ayush is taller than Urvashi and
Jindal (both) but not as tall as Mahi. Who among them is the third tallest?
• 1) Ayush
• 2) Jindal
• 3) Ketan
• 4) Urvashi
• 5) Can’t say
• Ans: 5
• Solution:
• Q3. In a company of 41 employees, L rank is 38th from the left end and S rank
is 17th from the right end. How many students are sitting in between L and
S?
• 1) 13
• 2) 14
• 3) Can’t be determined
• 4)1 2
• Ans: 4
• Solution:
• Here, the total number of persons is less than the sum of the position of L and the
position of S.
• Formula: Number of persons between that two persons = (Sum of positions of two
different persons/things from both sides) – (Total number of persons/things) – 2
• Q4. J is 16 ranks ahead of K in a class of 33. If K’s rank is 7th from the last,
what is J’s rank from the top?
• 1) 11th
• 2) 10th
• 3) 12th
• 4) 18th
• Ans: 1
• Solution:
• Given that J is 16 ranks ahead of K and K’s rank is 7th from the last.
• Q5. There are some crows sitting in a row and all of them are facing north. C1
is sitting 24th from the left end and C2 is sitting 16th from the right end. C2
sits 20th to the left of C1. What is the total number of crows sitting in the row?
• 1) 18
• 2) 19
• 3) 21
• 4) 12
• 5) 26
• Answer: 2
• Solution:
• C1 is 24th from left end and C2 is 20th to left of C1. So, C2 is 4th from left end and
given 16th from right end, so total = (4+16) – 1 = 19
• Q6. In a row of boxes, B1 is 20th from the right end and B2 is eleventh from
the left end. If they interchange their positions, B2 becomes fifteenth from
the left end. How many Boxes are there in the row?
• 1) 34
• 2) 35
• 3) 30
• 4) 29
• 5) None of these
• Ans: 1
• Solution:
• = 20 + 15 -1 = 34
• Q7. In a packet, the number of sticks is twice that of red balls and all of these
are arranged in a row. There is a Red ball and its rank is 18th from the top.
Suppose there are 9 sticks ahead of Red ball. If the total articles in the packet
are 42, then find the number of balls behind Red ball in the ranking order?
• 1) 7
• 2) 6
• 3) 9
• 4) More than 7
• Ans- 5
• Solution:
• Again, Red ball rank is 18th from the top and there are 9 sticks ahead of Red ball.
• 1) 20
• 2) 23
• 3) 32
• 4) 15
• 5) 19
• Ans-2
• Solution:
• Given that, Seema interchanges her position with the one who is sitting 6th to the
right of Neha, so the position of Seema after interchange is (10+6) = 16th from the
left end and also she is 8th from the right end, so the total number of females
members is (16+8)-1 = 23
• Amongst five sons, A, B, C, D and E, each got different Property from their father. A
got more than C but less than D. D got 85 properties. B got less than only E. The
one who got the minimum got 60 properties and the one who scored the highest,
got 91 properties.
• 1) B
• 2) A
• 3) C
• 4) E
• 5) Can’t be determined
• Ans: 1
• Solution:
• 1) B
• 2) A
• 3) D
• 4) E
• 5) Can’t be determined
• Ans: 2
• Solution:
• Level 3:
• Directions (1-2): Read the following information carefully to answer the
questions given below.
• Among Priti, Garima, Radha, Surya and Tanishq. Priti is two times younger than
Radha. Priti is younger than only two persons. Surya is older than Garima. Radha
is three times older than Tanishq. Surya is two times older than Tanishq.
• 1Q. If Surya is 10 years old, then find out whom age will be 30 years?
• 1) Priti
• 2) Radha
• 3) Tanishq
• 4) Garima
• 5) Can’t be determined
• Ans: 1
• Solution:
• 1) Tanishq
• 2) Radha
• 3) Surya
• 4) None of these
• 5) Can’t be determined
• Ans: 4
• Solution:
• 3Q. In a row of Pigeons, all are facing north, P1 is 9th from the left end and
P2 is 12th from the right end. There are three ministers between P1 and P2.
P1 and P3 are equidistant to P4. Find how many Pigeons are there in the row?
• 1) 15
• 2) 20
• 3) 4
• 4) 18
• 5) Can’t be determined
• Ans: 5
• Solution:
• Q4. In an Almirah, there are some blankets and no blanket is used more than
one time. There are 5 blankets between N and S, 8 blankets are there
between S and Q, If only 4 blankets are there before Q and 18 blankets are
thereafter N. then find out how many blankets are there?
• 1) 15
• 2) 27
• 3) 26
• 4) 38
• 5) Can’t say
• Ans: 3
• Solution:
• Six friends went for running– M, N, O, P, Q and R run at a different speed. Q runs
slower than P but is not the slowest. M runs faster than P but slower than N. The
one who runs the second fastest runs at the speed of 52 km/hr. The one who runs
the second slowest runs at the speed of 30 km/hr. The running speed of O is more
than 52 km/hr.
• Q5. Who among the following has the running speed less than 30 km/hr?
• 1) R
• 2) N
• 3) O
• 4) P
• 5) Can’t be determined
• Ans: 1
• Solution:
• Q6. Who among the following has the running speed 52 km/hr?
• 1) N
• 2) O
• 3) R
• 4) P
• 5) Can’t be determined
• Ans: 1
• Solution:
• Q. Who among the following has picked the second least quantity of Coconut?
• 1) E
• 2) B
• 3) C
• 4) None of these
• 5) Can’t be determined
• Ans: 1
• Solution:
• It is given that,
• Q8. Six participants named Arti, Sandhya, Hema, Jyoti, Lokesh and Kashira
won different monetary prizes in a competition. Kashira did not win the least
money among them. Arti got one third money of Sandhya and four times
money of Kashira’s money. The total money won by Lokesh and Kashira is
equal to the total money won by Arti and Hema. The difference in money
received between Hema and Lokesh is equal to the money which is won by
Jyoti.
• Q. If Kashira won 15000, then what will be the possible money won by Jyoti?
• 1) 6000
• 2) 17000
• 3) 49000
• 4) 45000
• 5) 65000
• Ans: 4
• Solution:
• It is given that,
• Lokesh + Kashira = Arti + Hema (equation 1); and Hema – Lokesh = Jyoti
• After, putting the value of Arti=4Kashira and Hema = Lokesh + Jyoti in equation 1,
we get;
• 3 Kashira = Jyoti
• If Kashira won 15000, then the possible money won by Jyoti will be (3×15000) =
45000
• In a School, there are six teachers named P, Q, R, S, T, and V. Each of them teaches a
different subject, i.e. Biology, Zoology, Music Instrument, History, Math and
Chemistry. Each of the teachers gets a different salary.
• R gets second highest salary and his teaching subject is neither Chemistry nor
Music-instrument. T gets the least salary among them and his teaching subject is
neither Math nor Biology. The teaching subject of S is Zoology and he gets more
salary than the salary of Q and V but less than the salary of P. The teaching subject
of Q is History and his salary is less than the salary of V. The teaching subject of R
is not Biology. The teacher whose teaching subject is Music instrument gets more
salary than the one who teaches Biology.
• Solution (9-10):
• P (Music) > R (Math) > S (Zoology) > V (Biology) > Q (History) > T (Chemistry)
• 1) Biology
• 2) Chemistry
• 3) Math
• 4) Zoology
• 5) None of these
• Ans: 1
• Solution:
• Q10. Who among the following has the third highest salary?
• 1) Q
• 2) S
• 3) T
• 4) P
• 5) None of these
• Ans-2
• Solution:
• Alphabet Test
•
• Important Facts About Letters :
• 1) Position of letters :
• Practice Questions :
• Que 1. Which of the following words cannot be formed using the letters of
the word ‘IMMEDIATELY’?
A ) DIET
B ) LATE
C ) LATELY
D ) MEDIATE
• Solution:
The word ‘LATELY’ cannot be formed using the letters of the word ‘IMMEDIATELY’.
Hence LATELY is correct.
• Que 2. How many meaningful words can be formed using the first, second,
sixth, and seventh letters of the word ‘INFORMATION’?
A ) None
B ) One
C ) Two
D ) More than two.
• Solution:
The letters are as follows: I, N, M, and A
Only one word can be formed using the above letters – MAIN.
Hence, the correct answer is One.
• Que 3. Which word can not be formed from the word ‘INTERROGATION?
A ) Gate
B ) Interrogate
C ) Oration
D ) Notation
• Ans: B ) Interrogate
• Solution:
Interrogate cannot be formed from the word Interrogation, as it requires an extra
‘e’ to complete the spelling.
Hence Interrogate is correct.
• Que 4. Direction: From the given alternative words, select the word which
cannot be formed using the letters of the given word UNIFORMITY?
A ) FORM
B ) FAMILY
C ) UNITY
D ) NORM
• Ans: B ) FAMILY
• Solution :
In Family, there is no A or L in the given Key Word “UNIFORMITY”
• Que 5. Which word will be 2nd last if all are arranged alphabetically as in a
dictionary?
A ) Analytics
B ) Analysis
C ) Analyst
D ) Arithmetic
• Ans: A ) Analytics
• Solution:
The arrangement as in a dictionary:
Analysis
Analyst
Analytics
Arithmetic
Hence, the correct answer is Analytics.
• Que 6. Find the word which can be made from letters of the given word
TRUTHFULNESS
A ) Funny
B ) Fruit
C ) Ruthless
D ) Serene
• Ans: C ) Ruthless
• Solution:
The word ‘Funny cannot be made because TRUTHFULNESS’ does not contain ‘Y’.
‘Fruit’ cannot be made because TRUTHFULNESS’ does not contain l’
and ‘Serene’ cannot be made because TRUTHFULNESS’ does not have 3 ‘E’.
Therefore, Ruthless is the correct answer.
• Que 7. Find the word which can be made from letters of the given word.
ELASTICITY
A ) ECSTASY
B ) ELONGATE
C ) TITULAR
D ) STATIC
• Ans: D ) STATIC
• Solution:
‘ECSTASY’ cannot be formed using ‘ELASTICITY’ because the word it has two ‘S’.
‘ELONGATE’ can also not be formed using ‘ELASTICITY’ because it contains ‘Gʻ
which is not present in ‘ELASTICITY.
‘TITULAR’ can also not be formed because ‘ELASTICITY’ does not contain ‘U’.
Hence, the correct answer is Static.
• Que 8. Find the word which cannot be made from letters of the given word.
FERTILITY
A ) TILE
B ) EAT
C ) LITTER
D ) FRET
• Ans: B ) EAT
• Solution:
The word ‘EAT’ cannot be made using the word ‘FERTILITY’ because the word does
not contain ‘A’.
Hence, the correct answer is EAT.
• Que 9. Find the word which cannot be made from letters of the given the
word, POPULARITY.
A ) PLURAL
B ) POPULAR
C ) TRAIL
D ) PURITY
• Ans: A ) PLURAL
• Solution :
The word ‘PLURAL’ cannot be formed using the word ‘POPULARITY’ as the word
requires two ‘L’ while ‘POPULARITY’ contains a single ‘L’.
Hence, the correct answer is PLURAL.
• Que 10. Find the word which cannot be made from letters of the given the
word, INTERACTION.
A ) INTERN
B ) TRACTION
C ) INTEND
D ) CARE
• Ans: C ) INTEND
• Solution:
The word ‘INTEND’ cannot be made using the letters of the given word because
the word ‘INTERACTION’ does not contain a ‘D’.
Hence, the correct answer is INTEND.
• Que 11. Which of the following word cannot form using letters of the word
‘Antagonises?
A ) Noise
B ) Noun
C ) Gate
D ) Gain
• Ans: B ) Noun
• Solution:
Clearly, ‘Noun’ cannot form using letters of the word ‘Antagonises’.
• Que 12. From the following alternatives select the word which can’t be
formed by using the letters of the word.
RAGHAVAN
A ) RAGHAV
B ) RANG
C ) GRAND
D ) VAN
• Ans: C ) Grand
• Solution:
The word ‘RAGHAVAN’ does not have a ‘D’ letter so the word ‘GRAND’ can’t be
formed.
•
• Que 13. If all letters in the word “PAINTING” are rearranged as they appear
in the English alphabet series then the position of how many letters will
remain unchanged after the rearrangement?
A ) 4
B ) 2
C ) 1
D)3
• Answer: C ) 1
• Solution :
The given word ‘PAINTING” and the rearrangement of its letters:
• P A I N T I N G
A GIIN NPT
• Observing the above arrangement, we can clearly see that position of only I will
remain unchanged.
• Que 14. What will be the 3rd letter of the meaningful English word formed
using 1st, 6th and 8th letters of the word “TUTORIAL”. Each letter is to be
used only once in the meaningful word. If more than 1 such word can be
formed then mark ‘M’ as your answer, if no such word can be formed then
mark ‘Z’ as your answer.
A ) T
B ) I
C ) M
D)Z
• Ans: A ) T
• Solution :
1st, 6th and 8th letters of the word “TUTORIAL” are T, I, and L.
A meaningful English word formed is ‘Lit’, which means “illuminated”.
Thus, the 3rd letter of ‘Lit’ is T.
Hence, T is the correct answer.
•
• Que 15. Directions: If each letter of the following words is written in English
alphabetical order and all new formed words are arranged in dictionary
order, then which of the following word will come first in dictionary order?
ORNAMENT ORACLE OBESITY ORBITAL TOWEL
A ) ABILORT
B ) ELOTW
C ) AEMNNORT
D ) BEIOSTY
• Ans: A ) ABILORT
• Solution :
Each letter of each word in English alphabetical order:
ORBITAL – ABILORT
ORNAMENT – AEMNNORT
TOWEL – ELOTW
ORACLE – ACELOR
OBESITY – BEIOSTY
The arrangement as per the dictionary: ABILORT, ACELOR, AEMNNORT, BEIOSTY,
ELOTW.
Hence ABILORT is correct.
• Que 16. How many such pairs of letters are there in the word ‘SCIENTIST’
each of which has as many letters between them in the word as in English
alphabetical order (both in forward and backward direction)?
A ) 1
B ) 4
C ) 2
D)3
• Ans: C ) 2
• Solution:
In the word ‘SCIENTIST’ we get the following pairs:
CE- 1 letter between C and E.
ST-no letter between Sand T.
Thus, we get 2 pairs.
Hence option C is correct.
• Que 17. What will be the 2nd letter of the meaningful English word formed
using 2nd, 5th, 7th and 8th letters of the word “HARDWARE”. Each letter is to
be used only once in the meaningful word. If more than 1 such word can be
formed then mark ‘L’ as your answer, if no such word can be formed then
mark ‘Z’ as your answer.
A ) Z
B ) A
C ) E
D)L
• Ans: D ) L
• Solution:
2nd, 5th, 7th and 8th letters of the word “HARDWARE” are A, W, R and E.
Only 2 meaningful English words are formed i.e. Ware (articles for sale)’, ‘Wear
(something on one’s body as clothing)’.
Hence option D is correct.
• Que 18. How many meaningful English words can be formed using 1st, 4th,
5th and 6th letters of the word “TECHNIQUES”. Each letter is to be used only
once in the meaningful word.
A ) 2
B ) 3
C ) 1
D ) 4
Ans: A ) 2
• Solution:
1st, 4th, 5th and 6th letters of the word “TECHNIQUES” are T, H, N and I.
Meaningful English words formed are:
Hint: Suggesting something in an indirect way.
Thin: having less fat
Hence, option A is correct.
• Que 19. If all letters in the word “CONSCIOUSNESS” are rearranged as they
appear in the English alphabet series then the position of how many letters
will remain unchanged after the rearrangement?
A ) 2
B ) 4
C ) 5
D)1
• Ans: B ) 4
• Solution:
The given word “CONSCIOUSNESS” and the rearrangement of its letters:
C O N S C I O U S N E S S
C C E I N N O O S S S S U
Observing the above arrangement, we can clearly see that the positions of C, O, S,
and S will remain unchanged.
• Que 20. What will be the 4th letter of the meaningful English word formed
using the 2nd, 3rd, 7th, 8th and 10th letters of the word “HORIZONTALlY”?
Each letter is to be used only once in the meaningful word. If more than 1
such word can be formed then mark ‘X’ as your answer, if no such word can
be formed then mark ‘Y’ as your answer.
A ) O
B ) R
C ) X
D)Y
• Ans: D ) Y
• Ans: The 2nd, 3rd, 7th, 8th, and 10th letters of the word “HORIZONTALLY” are O,
R, N, T and L.
No meaningful English word is formed.
Seating Arrangement
Seating arrangement is the arrangement of people/objects logically. This concept involves
the arrangement of people in many possible ways.
In these type of questions, you will have to arrange a group of persons satisfying certain
conditions. The questions on this topic can be asked in any sequence (linear arrangement,
circular arrangement). By applying the logical analysis, we can perform the logical
arrangement to answer the questions or decode.
The questions on Seating arrangement are the most common types of questions asked in
reasoning in all entrance exams. There are different types of problems with the concept of
seating arrangement as follows.
1. Linear Arrangement
2. Square/Rectangular Arrangement
3. Circular Arrangement
4. Triangular arrangement
5. Hexagonal arrangement
6. Pentagon arrangement.
Examples :
Following are some questions on seating arrangement as follows.
Example1 :
B, L, M, N, P, and Q are in a row. P and Q are in the center, B and L are at the ends. M is
sitting to the left of B. Who is to the right of L?
Solution –
The seating arrangement is as follows.
L–N–P–Q–M–B
Example2 :
Five boys are sitting to be photographed. S is to the left of R and to the right of B. M is to the
right of R. E is between R and M.
Solution –
The seating arrangement is as follows.
B–S–R–E–M
Example3 :
Five girls are standing in a line. One of the two girls at extreme end is P and the other is B. A
is to the right of S. C is to the immediate left of the B. What is the position of A from the left?
Solution –
The seating arrangement is as follows.
P–S–A–C–B
Seating Arrangement Reasoning Questions and Answers
Q1. Seven friends are sitting on a circular table facing the center. A sits second to the left of
B. C sits between B and G. D sits second to the left of C. E who sits second to the right of G
sits between A and F. Who sits second to the left of G?
a) A
b) B
c) C
d) E
Answer: b) B
Q2. In a rectangular table, A, B, C, and D are sitting on one side facing E, F, G, and H on the
other side. A is opposite to E and B is opposite to F. And G is opposite of C. Who is opposite
to D?
a) G
b) H
c) E
d) F
Answer: b) H
Q3. Six friends – A, B, C, D, E, and F – are sitting in a straight line facing North. A is sitting
fourth to the left of B and first to the right of F. D is sitting to the immediate left of C, who is
not sitting at either of the ends. Who is sitting at the extreme right end of the line?
a) A
b) B
c) E
d) F
Answer: b) B
A circular Arrangement is a type of Sitting Arrangement, where objects are placed around
a circle either facing the center or facing the direction opposite to center. One can see the
different types of questions on this topic. So it is highly advisable to practice an ample
amount of questions.
Direction (1-5): Study the following information carefully and answer the given questions.
A certain number of people sit around a circular table and face towards the center of the
table. Some of them like different colors. Three people sit between P and the one who
likes Grey. The one who likes Orange sits third to the left of Q. The number of people
sitting between P and Q is the same as the number of people sitting between Q and R. S
sits third to the right of R who doesn’t like Purple. Q sits second to the left of the one who
likes Grey. Two people sit between S and the one who likes Purple. The one who likes
Green sits fourth to the right of the one who likes Grey. Five people sit between T and the
one who likes Black. S doesn’t sit adjacent to the one who likes Black. Three people sit
between R and the one who likes Black. The one who likes Pink sits third to the right of the
one who likes Green. T doesn’t sit adjacent to the one who likes Pink. U sits to the
immediate left of the one who likes Black and third to the left of the one who likes Yellow.
The one who likes Red sits second to the right of the one who likes Yellow. One person sits
between the one who likes Green and T who is not a neighbour of S. The Same number of
people sit between the one who likes Grey and V from both sides.
Solution:
Three people sit between P and the one who likes Grey.
Here, we have two possible cases i.e. Case 1 and Case 2:
Q sits second to the left of the one who likes Grey.
The one who likes Orange sits third to the left of Q.
The number of people sitting between P and Q is the same as the number of people sitting
between Q and R.
Case 1.
U sits to the immediate left of the one who likes Black and third to the left of the one who
likes Yellow.
The one who likes Red sits second to the right of the one who likes Yellow.
The same number of people sit between the one who likes Grey and V from both sides.
Here, we have the final arrangement:
Case 2.
Que 1. How many people sit around the circular table?
a) Twenty-two
b) Eighteen
c) Twenty
d) Twenty-one
e) None of these
Que 3. Who amongst the following sits eighth to the right of the one who likes Red?
a) T
b) The one who likes Pink
c) The one who likes Grey
d) Q
e) None of these
Answer: Option (c)
Que 4. How many people sit between U and the one who likes Orange when counted from
the right of U?
a) Six
b) Eight
c) Five
d) Seven
e) None of these
Answer: Option (d)
Direction (6-10): Study the following information carefully and answer the given
questions.
Eight persons – A, B, C, D, E, F, G, and H are sitting around a circular table, but not
necessarily in the same order. Some of them are facing inside while the rest of them are
facing outside. Each of them likes different engineering branches viz. Agriculture,
Metallurgical, Automobile, Electrical, Marine, Petroleum, IT, and Aeronautics. F does not
sit immediately next to A. Only two persons sit between G and H. Immediate left of A likes
Metallurgical, who faces opposite to center. F and E are facing the same side and only two
persons are sitting between them. The one who likes Petroleum sits immediately next to
the one who likes IT but does not face the one who likes automobiles. E sits second to the
left of A. The one who likes Marine faces the one who likes Aeronautics. D sits diagonally
opposite B who sits to the immediate right of E and both E and B are facing the same
direction. G is facing the same direction as E. The one who likes automobiles sits between
F and B. F does not like Agriculture. A and C are facing outward direction but not as F. C sits
second to the left of G. The one who likes Aeronautics sits second to the right of the one
who likes automobiles. D is facing the same direction as F. G does not like automobiles.
Solution:
Que 8. Which of the following is not true regarding the given arrangement?
a) A sits immediately next to H.
b) G and C face the same direction.
c) D faces the person who likes Agriculture.
d) E and G sit opposite to each other
e) All of the above
Que 9. Who among the following person faces the one who sits third to the left of D?
a) A
b) D
c) E
d) G
e) Can’t be determined
Answer: Option (a)
Que 10. Four of the following are alike in a certain way and form a group. Which of the
following does not belong to that group?
a) F
b) D
c) H
d) E
e) B
Six people Jeevan, Kumar, Lalit, Manoj, Naveen, and, Omkar like different games i.e. Pubg,
Fortnite, temple run, Super Mario, cricket, and Car Race. They all have different brands of
mobiles i.e. Lava, Infinix, Vivo, Oppo, Nokia, and Motorola. All information is not necessarily
in the same order.
Kumar has Oppo’s mobile. Manoj likes Fortnite. Lalit does not like temple run and Super
Mario. Naveen does not have Motorola and lava. Jeevan likes the Car Race and has Nokia.
Omkar does not like Temple Run and PUBG. Manoj does not have Lava and VIVO. The one
who likes Temple Run does not have Vivo. Naveen does not like temple run, Pubg, and Super
Mario. The one who has Oppo does not like Super Mario. The one who likes Super Mario has
Vivo.
(b) Lalit
(c) Naveen
(d) Omkar
(e) Manoj
Ans: E
(b) Manoj-Pubg-Lava
(c) Omkar-Mario-Vivo
Ans: C
Person Game Mobile
Ans: B
(a) Nokia
(b) Lava
(c) Infinix
(d) Motorola
(e) Oppo
Ans: A
(d) Fortnite
(e) Cricket
Ans: E
Blood Relations
While solving the questions on blood relationship, one should be familiar with basic
concepts like various relations.
Important Terms:
Children of the same parent - Siblings
The easiest and non-confusing way to solve blood relationship problems would be to draw a
family tree diagram and increase the levels in the generation wise as shown below-
Below Examples illustrate the process for solving this kind of problems:
Example 1:
Pointing to a man in the photograph a woman said, “He is the brother of the daughter of the
wife of my husband” How is the man related to the woman?
Solution:
My husband means woman’s husband, wife of my husband means again women itself,
daughter of the wife of my husband means woman’s daughter and brother of the daughter
means woman’s son. So, the woman is the mother of the man in the photograph.
Example 2:
Introducing Sanghoti to guests, Dinabandhu said, “Her father is the only son of my
father”.How is Sanghoti related to Dinabandhu?
(a) Daughter
(b) Mother
(c) Sister
(d) Niece
Solution:
Example 3:
A woman introduces a man as the son of the brother of her mother. How is a man related to
the woman?
(a) Nephew
(b) Son
(c) Cousin
(d) Uncle
Solution:
Hence, man is woman’s Cousin.
Coding-Decoding
Coding decoding is an important part of the Quantitative Aptitude section in most of the
competitive examinations in India. Coding and decoding questions are a common feature in
the reasoning ability section of major Government exams such as those conducted by banks,
insurance companies, SSC, and RRB. These types of questions test a candidate’s ability to
decode a given condition or code presented in the statement and then answer the related
questions based on it.
Coding-Decoding Reasoning
Question – 01
Direction-
Ans: D
Solution:
Rank – vs
Entry – ke
Class – on
Blew – cs
Care – dn
Open – ut
Que – ed
Input/adjust – ai/ax
Hence, Option D is correct.
Question – 02
Direction-
In a certain code language,
A. care
B. blew
C. que
D. class
E. None of these
Ans: C
Solution:
Rank – vs
Entry – ke
Class – on
Blew – cs
Care – dn
Open – ut
Que – ed
Input/adjust – ai/ax
Hence, Option C is correct.
Question – 03
Direction-
In a certain code language,
A. on eu
B. hn cs
C. wb hj
D. dn on
E. None of these
Ans: B
Solution:
Rank – vs
Entry – ke
Class – on
Blew – cs
Care – dn
Open – ut
Que – ed
Input/adjust – ai/ax
Hence, Option B
Question – 04
A. P3#
B. E4@
C. 03@
D. P7#
Ans: D
Solution:
The letter in the code is the 2nd succeeding letter (in alphabetical series) of 2nd letter of the
word.
The symbol in the code is on the basis of the number of vowels in the word:
If there are even numbers of vowels, then the code will be @
Example:
“abstain”
2nd letter of the word is “B” and its 2nd succeeding letter in the alphabetical series is “D”.
Question – 05
In a code language,
‘purity abstain culture surpass’ is coded as ‘W5@ D4# W4@ W4#’
B. Turbine
C. Private
D. Hours
E. None of the above
Ans: B
Solution:
The letter in the code is the 2nd succeeding letter (in alphabetical series) of 2nd letter of the
word.
The symbol in the code is on the basis of the number of vowels in the word:
Example:
“abstain”
2nd letter of the word is “B” and its 2nd succeeding letter in the alphabetical series is “D”.
There are four consonants in the word, so the number is 4.
LETTER CODING
Letter Coding is a type in which the letters are replaced with other letters.
Take an example of Letter Coding:
JUNE is coded as “HSLC”, then what should be the code for JULY.
Explanation: To solve these kinds of problems, you have to remember that every alphabet
has a specific number.
So, in the same way, to code “JULY”, the same number should be decreased.
NUMBER CODING –
If “GEEKS” is coded as 35542, and GAME is coded as 3895, then what would be the code
for SKEEG?
So, here The code of every letter is already specified in the question itself, so no need to use
fixed codes of the letters.
Now, specify the number of each letter to solve the problem. If you observe the two words,
some of the letters are repeated, so no need to write the repeated letters.
Now, code the letters.
Q1. If EARTH is written as FCUXM in a certain code. How is MOON written in that code?
Solution :
Q2. If DELHI is written as EDMGJ in a certain code. How is NEPAL written in that code?
Solution :
Q3. If SYMBOL is written as NZTMPC is it a certain code? How is NUMBER written in that
code?
Solution:
Acc. to question
Acc. to question
Q5. In a certain code, NEWYORK is written as 111, how is NEWJERSEY written in that code?
Solution :
In NEWYORK
N = 14, E = 5, W = 23, Y = 25, O = 15, R = 18, K = 11
Total = 14 + 5 + 23 + 25 + 15 + 18 + 11 = 111
In NEWJERSEY
Total = 14 + 5 + 23 + 10 + 5 + 18 + 19 + 5 + 25
= 124
Q6. In a certain code, HARYANA is written as 8197151, how is DELHI written in that code?
Solution:
We used the number of alphabets here.
H=8
A=1
R = 18 = 1+8 = 9
Y = 25 = 2+5 = 7
For DELHI
D =4
E=5
L = 12 = 1+2 = 3
H=8
I=9
Hence, DELHI is written as 45389.
Q7. In a certain code BOMB is written as 5745 and BAY is written as 529, how is BOMBAY
written in that code?
Solution:
Acc. to question
Use the numbers to relate the words
Substitution Coding
It is a type of puzzle-solving that involves substituting specific words for other names.
Q1. If in a certain language, MADRAS is coded as NBESBT, how is BOMBAY coded in that
code?
(a) CPNCBX (b) CPNCBZ (c) CPOCBZ (d) CQOCBZ (e) None of these
answer - (b)
Explanation
Q2. In a certain code, TRIPPLE is written as SQHOOKD. How is DISPOSE written in that
code?
(a) CHRONRD (6) DSOESPI (c) ESJTPTF (d) ESOPSID (e) None of these
Answer - (a)
Explanation
Q3. If in a code language. COULD is written as BNTKC and MARGIN is written as LZQFHM,
how will MOULDING be written in that code?
(a) CHMFINTK (6) LNKTCHMF (c) LNTKCHMF (d) NITKHCMF (e) None of these
Answer: c
Q4. In a certain code, MONKEY is written as XDJMNL. How is TIGER written in that code?
(a) QDFHS (6) SDFHS (c) SHFDQ (d) UJHFS (e) None of these
Answer: a
(a) NQPTJHOJ (6) NQPTJOHI (c) NQTPJOHJ (d) NQPTJOHJ (c) None of these
Answer: d
NUMBER CODING –
If “GEEKS” is coded as 35542, and GAME is coded as 3895, then what would be the code
for SKEEG?
So, here The code of every letter is already specified in the question itself, so no need to use
fixed codes of the letters.
Now, specify the number of each letter to solve the problem. If you observe the two words,
some of the letters are repeated, so no need to write the repeated letters.